Está en la página 1de 150

Endocrinología 

Banco 

Pregunta 1 de 253 
Hombre  de  35  años,  de  profesión  carpintero,  previamente  sano. 
Hace 5 meses, durante contacto casual con un médico se detectaron 
rasgos  clínicos  de  crecimiento  acral,  que  el  paciente  no  había 
notado.  Al  interrogatorio  dirigido  manifestó  piel  grasa,  cefalea, 
tono de voz grave, aumento de la talla del calzado y apnea durante 
el sueño. Negó alteraciones de la visión EF. Peso 96.5 kg, Talla 1.70, 
IMC  33,39,  TA  150/90,  FC  63,  FR  19.  Hábitus  acromegálico, 
preeminencia  ciliar,  campimetría  por  confrontación  con 
hemianopsia  bitemporal,  macroglosia,  diastemia,  cuello 
comedones  y  acrocordones,  acantosis  nigricans,  ruidos  cardiacos 
rítmicos, frecuencia disminuida, sin soplos, campos pulmonares sin 
alteraciones,  extremidades  con  ensanchamiento  de  manos  y  pies. 
Laboratorio: glucosa 89, creatinina 0.9, colesterol 148, HDL 33, LDL 
97, albúmina 4.3, Na 148, K 4.03, Cl 108, Hb 15.8, plaquetas 156mil, 
leucocitos 4900. GH 9.4, IGF1 985 (109‐284), Cortisl 12, ACTH 45.4, 
prolactina 16.91. T3 1.2 (0.8‐2.0), T4 6.68 (5.1‐14.1), TSH 1.43, LH 4.1, 
FSH  5.3,  Testosterona  1.81  (1.42‐9.23)  ¿Cuál  es  la  causa  de 
hemianopsia bitemporal? 

 Compresión del quiasma óptico (10 Puntos) 
 Afección de la retina 
 Hemorragia ocular 
 Infiltración del nervio óptico 
 Isquemia hipofisaria. 

Respuesta correcta: Compresión del quiasma óptico. Los pacientes 
con macroadenomas hipofisarios pueden presentar alteraciones en 
la visión en 10% de los casos, debido a compresión o desviación del 
quiasma óptico. La manifestación característica en la hemianopsia 
unilateral  o  bilateral  temporal.  Otros  síntomas  incluyen  diplopia, 
alteraciones  de  la  percepción  de  profundidad  y  de  la  visión  de 
colores. Williams Textbook of endocrinology. Capitulo 9. Tumores 
hipofisarios. 
 
 
 

Pregunta 2 de 253 
Principal causa de este síndrome clínico: 

 Adenoma hipofisario productor de GH (10 Puntos) 
 Craneofaringioma 
 Idiopática 
 Diabetes 
 Secreción ectópica de GH 

Respuesta  correcta:  Adenoma  hipofisario  productor  de  GH.  La 


acromegalia es un síndrome clínico causado por exceso de hormona 
de  crecimiento.  En  >95%  de  los  casos  la  causa  es  un  adenoma 
hipofisario  secretor  de  GH.  El  resto  es  causado  por  tumores 
ectópicos  secretores  de  GHRH  o  GH.  Williams  Textbook  of 
endocrinology. Capitulo 9. Tumores hipofisarios. 
 
 
 
Pregunta 3 de 253 
Mecanismo fisiopatológico de crecimiento acral: 

 Efecto  directo  de  de  hormona  de  crecimiento  sobre  tejido 


conectivo. 
 Aumento de FGF23 
 Proliferación anormal de vasos sanguíneos en respuesta a GH 
 Aumento de todas las hormonas hipofisarias 
 Aumento de IGF1 producida en hígado, estimulada por exceso 
de GH. (10 Puntos) 

Respuesta  correcta:  Aumento  de  IGF1  producida  en  hígado, 


estimulada  por  exceso  de  GH.  El  exceso  de  producción  de  GH 
induce  un  aumento  en  la  producción  de  IGF1  en  el  hígado,  que 
ejerce efectos tróficos en los tejidos óseos y blandos nivel periférico. 
Williams  Textbook  of  endocrinology.  Capitulo  9.  Tumores 
hipofisarios. 
 
 
 

Pregunta 4 de 253 
Prueba de elección para el diagnóstico: 

 Cortisol urinario de 24 horas. 
 Prueba de estimulación con GHRH. 
 Medición de IGF1 aislada y prueba de tolerancia a la glucosa de 
2hrs con medición de GH. (10 Puntos) 
 Medición de GH aislada 
 Prueba de inhibición con 1mg de dexametasona nocturna. 
Respuesta  correcta:  Medición  de  IGF1  aislada  y  prueba  de 
tolerancia a la glucosa de 2hrs con medición de GH. El estándar de 
oro  para  el  diagnóstico  es  la  prueba  de  tolerancia  a  la  glucosa  y 
supresión de GH. Se mide GH basal, a los 30, 60, 90 y 120 minutos, 
se  considera  como  normal  GH  <1ng/ml  (método 
radioinmunometria), o <0.4ng/ml (método quimioiluminiscencia) a 
las  2  horas.  Williams  Textbook  of  endocrinology.  Capitulo  9. 
Tumores hipofisarios. 
 
 
 

Pregunta 5 de 253 
¿Cuál es el tratamiento de elección? 

 Cabergolina + RTE 
 Resección transesfenoidal. (10 Puntos) 
 Octreótide + radioterapia. 
 Bromocriptina + RTE 
 Octreótide 

Respuesta  correcta:  Resección  transesfenoidal.  El  tratamiento  de 


elección  es  la  cirugía  transesfenoidal.  Existen  varias  opciones  de 
tratamiento médico cuando el quirúrgico no es posible: Octreótide 
100mcg sc c/8hrs. Lanreótide 30mg cada 14 días. Cabergolina 0,5‐2 
mg  /sem.  Pegvisomant  10‐30mg/día  SC.  Williams  Textbook  of 
endocrinology. Capitulo 9. Tumores hipofisarios. 
 
 
 

Pregunta 6 de 253 
Mujer  de  32  años,  residente  de  Chiapas.  Antecedente  de 
hipertensión arterial sistémica controlada con captopril. Acude por 
amenorrea  de  3  meses,  malestar  general  intenso,  fatiga, 
disminución en la concentración, intolerancia al frío, ortostatismo y 
síncope.  Los  síntomas  iniciaron  de  forma  lenta,  después  de  su 
último parto. G5, P4,C1, AO. Refiere haber tenido preeclamspia y 
abruptio placentae, requirió cesárea y transfusión de 3PG y 1 pool 
plaquetario.  Inicialmente  notó  imposibilidad  para  la  lactancia 
materna,  no  se  realizó  abordaje.  Ha  recibido  tratamiento  con 
multivitamínicos sin mejoría. EF: TA 90/60 acostada, 80/50 de pie. 
FC  100x’  y  110x’  a  la  bipedestación.Peso  70kg,  Talla  1.63.  Piel  y 
mucosas  sin  hiperpigmentación.  Sin  bocio.  Disminución  de  vello 
axilar.  Taner  mamario  4,  Taner  genital  III.  Laboratorios:  T3  0.4 
ng/mL (0.64‐1.81), T4 3.5 ug/mL (5.91‐12.5), cT3 28% (32‐48.8), TSH 
0.5  (0.3‐5.0),  Tg  11.82ng/dL  (0‐36.8),  Cortisol  5.4  ug/dL  (6.7‐22.6), 
ACTH  10  pg/ml  (10‐100),  Estradiol  2.5  g/mL  (20‐148),  FSH  0.1 
mUI/mL (3.0‐20), LH 0.5mUI/mL (2.0‐15). ¿Cuál es el diagnóstico? 

 Insuficiencia suprarrenal primaria. 
 Panhipoituitarismo (10 Puntos) 
 Hipotiroidismo primario. 
 Falla ovárica prematura. 
 Deficiencia aislada del gonadotropo. 

Correcto 
 
 
 

Pregunta 7 de 253 
¿Cuál es la etiología en este caso? 
 Infecciosa. 
 Enfermedad infiltrativa 
 Síndrome de Sheehan . (10 Puntos) 
 Craniofaringioma. 
 Hipoxia neonatal. 

Correcto 
 
 
 

Pregunta 8 de 253 
¿Con qué hormona debe iniciarse el tratamiento de sustitución? 

 Etinil‐estradiol 
 Levotiroxina 
 Adrenalina 
 Hidrocortisona (10 Puntos) 
 Hormona del crecimiento 

Correcto 
 
 
 

Pregunta 9 de 253 
¿Cuál es un factor predisponente en esta paciente? 

 Edad 
 Múltiples embarazos 
 Antecedente de hipertensión (10 Puntos) 
 Lugar de residencia 
 Sobrepeso 

Correcto 
 
 
 

Pregunta 10 de 253 
Hombre  de  24  años.  Psicólogo.  AHF:  Abuelo  paterno:  vitiligo. 
Padre  y  hermana:  psoriasis.  Tíos  paternos  con  hipotiroidismo 
primario autoinmune. APNP: actividad física aeróbica y anaeróbica 
5  ves/semana,  consumo  de  suplementos  alimenticios.  COMBE 
negada. PA: hace 4 meses recibió tratamiento homeopático por acné 
(autotransfusión  intramuscular  glútea  de  6  ml  de  sangre)  y 
peróxido  de  benzoilo/clindamicina  tópico.  Tres  días  después 
presentó polidipsia con ingesta de 20L/d de agua fría, nicturia (x10), 
piel seca, intolerancia al ejercicio. EF: Talla 1.77m, Peso 83 kg, IMC 
26.49,  FC  50,  FR  16,  TA  110/70.  Conformación  mesomórfica,  sin 
facies  característica.  Bien  hidratado.  Cabeza  y  cuello  sin 
alteraciones. Campos visuales por confrontación normales. Cuello, 
tórax,  abdomen  y  extremidades  sin  alteraciones.  Tanner  V. 
Exploración  neurológica  sin  datos  anormales.  Laboratorios:  Hb 
18.2g/dL. Glu 84mg/dL, Creatinina 1.2mg/dL, Na 149 mEq/L, K 5 
mEq/L, Cl 110 mEq/L, Ca 10.2 mEq/L, P 4.4 mEq/L, Alb 4.7 g/dL, 
Osmolaridad  310  mOsm/mL,  NaU  28  mEq/L.  Densidad  urinaria 
1.007.  Proteinas  (‐).  pH  6.5,  NaU  28,  KU  18.6,  CrU  19.  El  cuadro 
clínico corresponde a: 

 Diabetes insípida (10 Puntos) 
 Diabetes mellitus 
 Síndrome de Sheehan 
 SIHAD 
 Ninguno de los anteriores. 

Correcto 
 
 
 

Pregunta 11 de 253 
¿Qué estudio es útil para confirmar su sospecha? 

 USG renal 
 Prueba de deshidratación. (10 Puntos) 
 IRM de cráneo. 
 Curva de tolerancia a la glucosa oral. 
 Medición de ácido úrico 

Correcto 
 
 
 

Pregunta 12 de 253 
En la prueba previa, el paciente logró aumentar su OsmU 70% con 
respecto  a  la  basal,  con  administración  de  desmopresina.  El 
diagnóstico es: 

 Diabetes insípda central parcial 
 Diabetes insípda nefrogénica parcial 
 Diabetes insípda central completa (10 Puntos) 
 Diabetes insípda nefrogénica completa 
 Polidipsia primaria. 

Correcto 
 
 
 

Pregunta 13 de 253 
¿Cuál es la causa más frecuente de la enfermedad del paciente?: 

 Nefrogénica 
 Secundaria (10 Puntos) 
 Genética 
 Idiopática 
 Vascular 

Correcto 
 
 
 

Pregunta 14 de 253 
¿En este caso cuál es el tratamiento de elección? 

 Insulina 
 Restricción hídrica 
 Hidratación  a  libre  demanda  +  Desmopresina  IN  o  VO  (10 
Puntos) 
 Hidratación a libre demanda 
 Vasopresina IV. 

Correcto 
 
 
 

Pregunta 15 de 253 
Hombre  de  26  años,  cuenta  con  el  diagnóstico  de  diabetes  tipo  1 
desde hace 7 años, fumador. Ha estado hospitalizado dos veces por 
crisis  hiperglucémicas.  No  se  han  estudiado  complicaciones 
crónicas.  Refiere  desde  hace  algunos  meses  parestesias  en  las 
plantas de los pies y dolor ardoroso. Utiliza insulina NPH 20UI en 
la  mañana  y  10  UI  en  la  noche.  Laboratorios  recientes:  Hba1c  de 
9.8%,  glucosa  270mg/dl,  creatinina  0.88  mg/dl,  colesterol  215 
mg/dL,  HDL  38,  LDL  121  mg/dL,  triglicéridos  190  mg/dL. 
Albumina en orina de 24 horas: 10mg/día. ¿Cuál es la complicación 
aguda más frecuente en esta enfermedad? 

 Pancreatitis 
 Neumonía 
 Estado hiperosmolar hiperglucémico 
 Infección urinaria 
 Cetoacidosis diabética. (10 Puntos) 

Correcto 
 
 
 

Pregunta 16 de 253 
¿Cuándo  se  recomienda  la  primera  revisión  oftalmológica 
endiabetes tipo 1? 

 A los 3 años del diagnóstico 
 A los 10 años del diagnóstico 
 A los 5 años del diagnóstico (10 Puntos) 
 No se requiere revisión oftalmológica 
 Al momento del diagnóstico 

Correcto 
 
 
 

Pregunta 17 de 253 
La fisiopatogenia de esta enfermedad es: 

 Secundaria a una mutación inactivante del receptor de insulina. 
 Secundario a una infección 
 Secundaria a destrucción autoinmune del páncreas (10 Puntos) 
 Secundaria a isquemia 
 Secundaria a mala alimentación 

Correcto 
 
 
 

Pregunta 18 de 253 
Como meta de prevención primaria, este paciente requiere: 

 Marcapasos 
 Fibrato 
 IECA o ARA 
 Insulina ultralenta 
 Estatina (10 Puntos) 

Correcto 
 
 
 

Pregunta 19 de 253 
Diabetes  insulinodependiente  que  por  lo  general  se  presenta 
después de los 25 años y tiene etiología autoinmune: 

 Diabetes tipo 2 
 Diabetes por daño pancreático 
 Diabetes insípida. 
 Diabetes tipo LADA (10 Puntos) 
 Diabetes tipo MODY 

Correcto 
 
 
 

Pregunta 20 de 253 
Hombre  de  62  años  con  diabetes  mellitus  tipo  2  de  10  años  de 
evolución,  en  tratamiento  con  metformina  y  sitagliptina.  Hace 
ejercicio físico escaso y realiza una dieta adecuada. En los últimos 
6 meses ha perdido peso y tiene astenia. Sus controles glucémicos 
se han deteriorado pasando de glucemias basales de 110‐140 mg/dl 
a  glucemias  de  170‐200  mg/dl,  así  como  su  hemoglobina 
glucosilada que ha pasado de 7,1 a 8,5%. Pesa 60kg, mide 1.67. TA 
120/70. La medida terapéutica más adecuada a realizar es: 
 Sustituir metformina por glimepirida 
 Sustituir sitagliptina por pioglitazona 
 Asociar al tratamiento una dosis de insulina basal. (10 Puntos) 
 Asociar al tratamiento ascarbosa. 
 Aumentar  la  ingesta  de  proteínas  e  hidratos  de  carbono  de 
cadena larga en la dieta para mejorar la astenia y la pérdida de peso 

Correcto 
 
 
 

Pregunta 21 de 253 
La causa de fracaso al tratamiento es: 

 Mal apego al tratamiento farmacológico 
 Pérdida de la función de las células beta (10 Puntos) 
 Pérdida de receptores para sitagliptina 
 Mal apego a dieta 
 Deterioro renal 

Correcto 
 
 
 

Pregunta 22 de 253 
El esquema de insulina que elegiría para este paciente sería: 

 Bolos preprandiales de insulina regular 
 Basal – bolos con insulina NPH y rápida 
 Bolos preprandiales de insulina aspart 
 Dosis única de NPH o glargina. (10 Puntos) 
 No indicaría insulina 

Correcto 
 
 
 

Pregunta 23 de 253 
De  las  siguientes  insulinas,  elija  la  que  posee  el  efecto  más 
prolongado: 

 Glargina (10 Puntos) 
 NPH 
 Glulisina 
 Lispro 
 Regular 

Correcto 
 
 
 

Pregunta 24 de 253 
De las siguientes insulinas, elija la que posee el efecto más corto: 

 NPH 
 Regular 
 Detemir 
 Glargina 
 Lispr0 (10 Puntos) 

Correcto 
 
 
 

Pregunta 25 de 253 
Meta de colesterol LDL de acuerdo a sus factores de riesgo: 

 No existe una meta de tratamiento para este caso en particular 
 menor de 70 mg/dL 
 Entre 130 y 160 mg/dL 
 Entre 100 y 130 mg/dL 
 ‐ menor de 100 mg/dL (10 Puntos) 

Correcto 
 
 
 

Pregunta 26 de 253 
A  64  year‐old  retired  executive  with  a  history  of  type  2  diabetes 
mellitus  for  10  years  is  referred  to  you  to  discuss  risks  of 
atherosclerotic disease. She has been without symptoms of cardiac 
ischemia,  and  her  exercise  tolerance  is  normal.  She  follows  a 
regimen  to  limit  the  amount  of  carbohydrates  she  consumes  and 
takes metformina 2,500 mg per day. She also is on enalapril 10 mg 
per day (hypertension), 100mg aspirin and fenobifrate 160mg daily 
for hypetriglyceridemia. Her blood pressure is 128/74 mmhg, BMI 
27 kg/m2. The following laboratory tests were obtained: 
 ‐ Increase enalapril to 10mg/ twice daily 
 ‐ Addition of a statin (moderate doses) (10 Puntos) 
 ‐ Vitamin B supplements 
 Addition of sitagliptin 50mg per day 
 ‐15 units of NPH insulin at bedtime 

Correcto 
 
 
 

Pregunta 27 de 253 
What is her global risk for cardiovascular events: 

 ‐ Very low, because she is on good control of type 2 diabetes 
 High 
 Very high 
 ‐ Low, because she is postmenopausal women 
 ‐ Moderate (10 Puntos) 

Correcto 
 
 
 

Pregunta 28 de 253 
Which scrutiny tests she needs: 

 ‐ Computed tomography of heart 
 ‐ Diagnostic cardiac catheterization 
 ‐Echocardiogram and electrocardiogram 
 ‐ Polysomnography 
 ‐ Microalbuminuria (10 Puntos) 

Correcto 
 
 
 

Pregunta 29 de 253 
Which are the goals for LDL‐C? 

 ‐ The same that general population 
 ‐ Less than 70 mg/dL 
 ‐ less than 100 mg/dL (10 Puntos) 
 ‐ Between 101 and 130 mg/dL 
 ‐ between 131‐160 mg/dL 

Correcto 
 
 
 

Pregunta 30 de 253 
Time to start scrutiny for diabetic retinopathy: 

 ‐ When the familiar history is strong for retinopathy 
 ‐ At diagnosis (10 Puntos) 
 ‐ Only if there is changes in glucose control 
 ‐ Scrutiny is not necessary without symptoms 
 ‐ Five years after diagnosis 

Correcto 
 
 
 

Pregunta 31 de 253 
A  43‐year‐old  man  with  a  history  of  active  smoking,  chronic 
hypertension  and  type  2  diabetes  mellitus  treated  with  oral 
hypoglycemic  agents,  presents  at  emergency  room  with  fatigue, 
weakness,  weight  loss,  hyperpigmentation  of  skin  and  mucous 
membranes,  symptomatic  hypoglycemia  and  diffuse  abdominal 
pain, nausea, and vomiting, since 2 weeks ago. Physical exam: BP 
70/60, CR 110, 19 RR 29, lungs with bibasal crackles. Abdomen soft 
without peritoneal irritation. Laboratories: Glucose 58 19 BUN, Cr 
1.1, sodium 123 mEq / l, 5.3 meq potassium, 12 g Hb / dL Leukocytes 
19mil.  To  make  the  diagnosis  of  chronic  primary  adrenal 
insufficiency must be found: 

 osteoporosis, hypertension and hyperglycemia. 
 weakness, arterial hypertension and hypokalemia. 
 hyperpigmentation, hyperglycemia and hypokalemia. 
 hypokalemia, hyponatremia and orthostatic hypotension 
 hyperpigmentation, orthostatic hypotension and hyperkalemia. 
(10 Puntos) 

Correct  answer:  hyperpigmentation,  orthostatic  hypotension  and 


hyperkalemia.The  most  common  symptoms  of  adrenal 
insufficiency  include  progressive  weakness  and  fatigue, 
hypoglycemia,  weight  loss  and  gastrointestinal  symptoms 
(abdominal  pain,  nausea  and  vomiting).  The  primary  disease 
includes  involvement  mineralocorticoid  secretory  tissue  causing 
hipoaldosteronism  with  sodium  loss,  hyperkalemia,  avidity  for 
salt, orthostatic hypotension and mild metabolic acidosis. Lack of 
cortisol  produces  a  compensating  increase  in  ACTH  synthesis, 
leading  to  the  characteristic  skin  hyperpigmentation.  Williams 
Textbook of Endocrinology. Chapter 5. The adrenal cortex. 
 
 
 

Pregunta 32 de 253 
The main cause of adrenal insufficiency is 

 autoimmune disease (10 Puntos) 
 anticoagulation 
 tuberculosis 
 bilateral adrenal infarction. 
 trauma 

Adrenal  insufficiency  is  a  result  of  failure  of  the  adrenal  gland 
(primary), deficient secretion of ACTH by the pituitary (secondary) 
or  CRH  from  the  hypothalamus  (tertiary).  The  main  cause  of 
primary  adrenal  insufficiency  in  developed  countries  is  the 
autoimmune adrenalitis, while the primary cause of failure is high 
surprarrenal  abrupt  withdrawal  of  exogenous  glucocorticoids. 
Williams  Textbook  of  Endocrinology.  Chapter  5.  The  adrenal 
cortex. 
 
 
 

Pregunta 33 de 253 
The steroid of choice for treating adrenal crisis is: 

 betamethasone 
 hydrocortisone (10 Puntos) 
 prednisolone 
 methylprednisolone 
 dexamethasone 

Correct  answer:  hydrocortisone.  The  substitution  treatment  of 


Addisonʹs disease requires steroids with medium or short half‐life, 
such  as  hydrocortisone  or  prednisone.  Hydrocortisone  possesses 
mineralocorticoid action, so is considered the steroid of choice, plus 
adequate water replacement. Williams Textbook of Endocrinology. 
Chapter 5. The adrenal cortex. 
 
 
 

Pregunta 34 de 253 
A  drug  that  can  trigger  adrenal  crisis  in  patients  with  untreated 
adrenal insufficiency. 

 aspirin 
 hydrochlorothiazide 
 levothyroxine (10 Puntos) 
 lithium 
 amiodarone 

Correct  answer:  levothyroxine.  In  patients  with  concomitant 


hypothyroidism  and  adrenal  deficiency,  the  steroid  replacement 
must be the first step, since the administration of thyroid hormones 
may  precipitate  an  adrenal  crisis.  Williams  Textbook  of 
Endocrinology. Chapter 5. The adrenal cortex. 
 
 
 

Pregunta 35 de 253 
The effect of aldosterone in the kidney is: 

 inhibits the basolateral Na+ / K+ pump 
 inhibits the conversion of angiotensinogen to angiotensin I 
 increase the number of aquaporins in plasmatic membrane 
 opens the voltage‐dependent calcium channels 
 stimulates the basolateral Na+ / K+ pump (10 Puntos) 

Correct  answer:  stimulates  the  basolateral  Na+  /  K+  pump. 


Aldosterone acts on the renal tubular sodium transport in the distal 
convoluted tubule, by stimulating the basolateral Na+/ K+ pump, 
increasing  the  reabsorption  of  sodium  and  water;  and  increasing 
potassium excretion. Also converts angiotensinogen to angiotensin 
I.  Williams  Textbook  of  Endocrinology.  Chapter  5.  The  adrenal 
cortex. 
 
 
 

Pregunta 36 de 253 
A  42‐year‐old  obese  woman  with  type  2  diabetes,  diabetic 
retinopathy,  hyper¬tension,  and  hirsutism  presents  to  discuss  an 
elevated prolactin level of 144.8 ng/mL (normal range, 4.8 to 23.3 
ng/mL) found by her Ob‐Gyn two months ago. She complained of 
galactorrhea and no menses for one year. A repeat prolactin level 
was  also  elevated,  at  109  ng/mL.  The  following  are  causes  of 
hyperprolactinemia, EXCEPT: 

 chronic renal or hepatic failure 
 estrogen use 
 obesity (10 Puntos) 
 hypothyroidism 
 pregnancy 

Correct  answer:  obesity.  There  are  many  causes  of 


hyperprolactinemia.  Factors  that  can  increase  prolactin  secretion 
include  pregnancy,  nursing,  physiologic  stress,  estrogen  use, 
polycystic ovary syndrome, hypothyroidism, and chronic renal or 
hepatic  failure.  Head  trauma,  use  of  certain  medications 
(verapamil, neuroleptics, antipsychotics, and antidepressants), and 
presence  of  nonsecretory  sellar  or  suprasellar  masses  can  also 
increase prolactin levels. In general, signs and symptoms are due 
to either the effect of excess hormone secretion (ie, galactorrhea and 
amenorrhea)  or  local  compression  (ie,  new‐onset  or  persistent 
headache, dizziness, visual changes, and vision loss). A review of 
medications, including estrogen therapy, and history of fertility or 
gonadal  dysfunction  should  be  documented.  Elevated  prolactin 
levels can result in secondary hypogonadism. Williams Textbook 
of Endocrinology. Chapter 9.Pituitary masses and tumors. 
 
 
 

Pregunta 37 de 253 
The size of microadenomas does not exceed: 

 6mm 
 8mm 
 4mm 
 2mm 
 10mm (10 Puntos) 
Correct  answer:  10mm.  A  microadenoma  is  defined  as  an 
intrasellar  mass  less  than  10  mm  in  diameter.  A  macroadenoma, 
defined as larger than 10 mm in diameter, can cause enlargement 
of the sella turcica. Williams Textbook of Endocrinology. Chapter 
9.Pituitary masses and tumors. 
 
 
 

Pregunta 38 de 253 
The drug of choice for treatment of prolactinoma is: 

 leuprolide acetate 
 mestranol 
 danazol 
 cabergoline (10 Puntos) 
 tamoxifen 

Correct  answer:  cabergoline.  Optimal  treatment  outcomes  for  a 


prolactinoma include normalization of PRL levels (and associated 
signs  and  symptoms)  and  complete  tumor  removal  or  shrinkage 
with  a  reversal  of  tumor‐mass  effects.  Medical  management  of 
prolactinomas  with  dopamine  agonist  drugs  has  been  widely 
recommended  as  the  treatment  of  choice.  Bromocriptine  and 
cabergoline  lowers  elevated  PRL  levels,  restores  abnormal 
menstrual  function  in  80%  to  90%  of  patients,  shrinks 
prolactinomas,  restores  impaired  sexual  function,  and  improves 
galactorrea.  Cabergoline  has  a  longer  duration  of  action  than 
bromocriptine and is usually administered once or twice weekly. 
Since its introduction, it has surpassed bromocriptine as the first‐
line  therapeutic  choice  for  most  patients.  Williams  Textbook  of 
Endocrinology. Chapter 9.Pituitary masses and tumors. 
 
 
 

Pregunta 39 de 253 
The mechanism of action of the drug you chose is: 

 anti‐androgenic 
 dopaminergic agonists (10 Puntos) 
 GnRH analogues 
 dopaminergic antagonists 

Correct  answer:  dopaminergic  agonists.  Medication  therapy 


involves  treatment  with  a  dopamine  agonist,  which  directly 
inhibits prolactin secretion by the tumor and therefore suppresses 
tumor growth. The goal of medication therapy is to suppress the 
prolactin level to normal range and restore gonadal function. The 
two  dopamine  agonists  used  are  bromocriptine  and  cabergoline. 
Williams  Textbook  of  Endocrinology.  Chapter  9.Pituitary  masses 
and tumors. 
 
 
 

Pregunta 40 de 253 
What is the cause of hypogonadism in patients with prolactinoma? 

 destruction of gonadotropic cells 
 decreased production of GH 
 testicular atrophy 
 congenital defect 
 decreased production of GnRH (10 Puntos) 

Correct  answer:  decreased  production  of  GnRH.  Elevated  PRL 


causes  sexual  dysfunction  via  a  short‐loop  feedback  effect  on 
gonadotropin pulsatility that presumably inhibits GnRH as well as 
LH pulse frequency and amplitude. High PRL also directly inhibits 
ovarian  and  testicular  function.  Williams  Textbook  of 
Endocrinology. Chapter 9.Pituitary masses and tumors. 
 
 
 

Pregunta 41 de 253 
A 34‐year‐old woman presented with an asymptomatic enlarging 
nodule  of  the  right  thyroid  lobe.  Ultrasonography  revealed  a 
diffusely  enlarged  right  thyroid  lobe  with  three  hypoechogenic 
lesions,  the  largest  of  which  measured  5  cm  in  maximum 
dimension. Fine Needle Aspiration biopsy was performed, which 
showed  follicular  epithelial  cells,  with  optically  clear  nuclei  and 
nuclear  grooves,  suggestive  of  a  diagnosis  of  Papillary  thyroid 
carcinoma. Total thyroidectomy was performed. What is the risk of 
malignancy in thyroid nodules? 

 5% (10 Puntos) 
 No risk 
 20% 
 10% 
 1% 

Correct answer: 5%. A thyroid nodule is a lesion within the thyroid 
gland  that  is  distinguished  from  the  surrounding  thyroid 
parenchyma,  usually  are  incidental  findings.  The  prevalence  of 
palpable  thyroid  nodules  in  the  general  population  is  4%  in 
autopsy  series  50%  is  reached.  95%  of  thyroid  nodules  are 
benign.Williams Textbook of endocrinology. Chapter 14. Nontoxic 
Diffuse and Nodular Goiter and Thyroid Neoplasia. 
 
 
 

Pregunta 42 de 253 
Most common malignant tumor of the thyroid: 

 papillary cancer (10 Puntos) 
 Metastasis. 
 anaplastic cancer 
 medullary cancer 
 follicular cancer 

Correct answer: papillary cancer. Papillary carcinoma is the most 
common (60‐70%) thyroid malignant neoplasm, it has a mortality 
rate  3‐7%.  Papillary  cancer  is  histologically  characterized  by  the 
presence  of  pesudopapilas,  Psammoma  bodies,  and  nuclear 
grooves  pseudo  inclusions.  Williams  Textbook  of  endocrinology. 
Chapter  14.  Nontoxic  Diffuse  and  Nodular  Goiter  and  Thyroid 
Neoplasia. 
 
 
 

Pregunta 43 de 253 
The mutation associated with medullary thyroid carcinoma is 

 RAS 
 wolframina 
 RET (10 Puntos) 
 p53 
 menina 

Correct  answer:  RET.  Medullary  thyroid  carcinoma  accounts  for 


fewer  than  10%  of  thyroid  malignancies.  It  arises  from  the 
parafollicular or C cells of the thyroid gland, and the tumor cells 
typically  produce  an  early  biochemical  signal  (hypersecretion  of 
calcitonin).  The  tumor  occurs  in  both  sporadic  and  hereditary 
forms, the latter making up about 20% of the total. The hereditary 
variety can be transmitted as a single entity (familial MTC), or it can 
arise as part of MEN2A or MEN2B síndrome associated with the 
RET  proto‐oncogene.  Williams  Textbook  of  endocrinology. 
Chapter  14.  Nontoxic  Diffuse  and  Nodular  Goiter  and  Thyroid 
Neoplasia. 
 
 
 

Pregunta 44 de 253 
It is a finding in thyroid ultrasound suggesting malignancy: 

 absence of lymphadenopathy 
 decreased central vascularity 
 eggshell calcifications 
 microcalcifications (10 Puntos) 
 hyperechogenicity 

Correct  answer:  microcalcifications.  Attempts  have  been  made  to 


develop criteria for distinguishing benign and malignant nodules. 
Echo‐free  (cystic),  spongiform,  and  homogeneously  hyperechoic 
lesions  are  reputed  to  carry  a  low  risk  of  malignancy.  Positive 
predictive  criteria  of  malignancy  include  predominantly  solid 
nodules  and  absence  of  cystic  elements,  hypoechoic  nodules, 
presence  of  microcalcifications,  irregular  margins  and  absence  of 
halo,  and  a  taller  rather  than  a  wide  shape  measured  in  the 
transverse dimension. The color Doppler finding of predominantly 
internal  or  central  blood  flow  appears  to  increase  the  risk  that  a 
nodule is malignant.Williams Textbook of endocrinology. Chapter 
14. Nontoxic Diffuse and Nodular Goiter and Thyroid Neoplasia. 
 
 
 

Pregunta 45 de 253 
It is the most aggressive thyroid cancer: 

 lymphoma 
 anaplastic (10 Puntos) 
 papillary 
 medullary thyroid carcnioma 
 follicular 

Correct answer: anaplastic. Anaplastic carcinoma constitutes about 
1% to 2% of all thyroid carcinomas, usually occurs after the age of 
60 years, and is slightly more common in women. This carcinoma 
is  highly  malignant,  is  nonencapsulated,  and  extends  widely. 
Evidence of invasion of adjacent structures, such as skin, muscles, 
nerves, blood vessels, larynx, and esophagus, is common. Distant 
metastases occur early in the course of the disease in lungs, liver, 
bones, and brain. This carcinoma has the worst prognosis of thyroid 
cancers, treatment should be initiated rapidly to avoid death from 
locally  infiltrative  disease  and  possible  suffocation.  Treatment 
consists of surgical resection of the tumor tissue present in the neck, 
if  feasible,  followed  by  a  combination  of  external  irradiation  and 
chemotherapy.  Williams  Textbook  of  endocrinology.  Chapter  14. 
Nontoxic Diffuse and Nodular Goiter and Thyroid Neoplasia. 
 
 
 

Pregunta 46 de 253 
A  85‐year‐old  man,  resident  of  a  nursing  home  is  brought  to  the 
emergency room by decreased alertness, unresponsive to stimuli. 
He  has  diagnosis  of  diabetes  since  46  years  old,  treated  with 
glyburide  and  metformin.  Physical  examination:  unconscious, 
dehydrated.  BP  90/50,  CR  105c’,  RR  24x’,  T:  49.7ºC,  bibasilar 
pulmonar  crackles.  Labs:  143  Na,  2.5  K,  Gluc  675,  pH  7.37.  EGO: 
ketones +, Hb 19, Leu 18,500. ¿What is the most likely diagnosis?: 

 hyperosmolar hyperglycemic state (10 Puntos) 
 severe hypoglycemia 
 cerebral vascular event 
 myxedema coma 
 diabetic ketoacidosis 

Correct answer: hyperosmolar hyperglycemic state.HHS is one of 
the  acute  complications  of  diabetes  mellitus.  HHS  occurs 
predominantly in type 2 diabetes but can occur in type 1 most often 
affects the  elderly,  nursing home  residents  and  people  who  have 
free access to fluids. The pathophysiology is complex, it is because 
there  is  sufficient  circulating  insulin  to  inhibit  lipolysis  but  is 
insufficient  to  induce  the  peripheral  utilization  of  glucose. 
characterized  by  severe  hyperglycemia,  hyperosmolality,  and 
dehydration in the absence of significant ketoacidosis. Manual de 
terapéutica  médica  y  procedimientos  de  urgencias  INCMSNZ. 
Capitulo 35. Estado hiperosmolar hiperglucémico. 
 
 
 

Pregunta 47 de 253 
The first step in the treatment of this patient is: 

 sodium bicarbonate 
 hydration (10 Puntos) 
 antibiotics 
 flumazenil 
 insulin 

Correct  answer:  hydration.  The  mainstay  of  treatment  is  fluid 


administration:  SS  0.9  or  0.45%:  2‐3  L  in  the  first  2  h  and 
subsequently SS0.9% 1L every 2 h until blood glucose 250 mg / dL. 
Manual  de  terapéutica  médica  y  procedimientos  de  urgencias 
INCMSNZ. Capitulo 35. Estado hiperosmolar hiperglucémico. 
 
 
 

Pregunta 48 de 253 
A risk factor for developing this condition is: 

 to be resident of a nursing home (10 Puntos) 
 glibenclamide using 
 hypokalemia 
 renal failure 
 metformin 

Correct answer: to be resident of a nursing home. It occurs more 
frequently  in  elderly,  nursing  home  residents  or  people  who  are 
prone to dehydration. There are many predisposing factors, similar 
to  CAD.  To  EHH  the  most  important  is  dehydration.  Manual  de 
terapéutica  médica  y  procedimientos  de  urgencias  INCMSNZ. 
Capitulo 35. Estado hiperosmolar hiperglucémico. 
 
 
 

Pregunta 49 de 253 
What is the most likely underlying cause of this complication in this 
patient? 

 hyperglycemia 
 unknown 
 alcoholism 
 infection (10 Puntos) 
 lack of adherence to treatment 

Correct  answer:  infection.  The  patient  is  an  elderly  resident  of  a 
nursing  home,  which  confers  risk  of  HHS.  In  addition,  physical 
examination  reveals  the  presence  of  fever  and  pulmonary  rales, 
suggesting the presence of pneumonia as a trigger for HHS. 
 
 
 

Pregunta 50 de 253 
The diagnosis of this complication require the presence of: 
 pH <7.30 
 fever 
 urine ketones +++ 
 urinary osmolarity> 500 mOsm/l 
 plasma osmolarity >320mOsm/l (10 Puntos) 

Correct  answer:  plasma  osmolarity  >320mOsm/l.  The  diagnostic 


criteria for HHS are: glucose> 600, pH> 7.30, bicarbonate> 18meq / 
L  and  plasma  osmolality>  320mOsm  /  kg.  Manual  de  terapéutica 
médica  y  procedimientos  de  urgencias  INCMSNZ.  Capitulo  35. 
Estado hiperosmolar hiperglucémico. 
 
 
 

Pregunta 51 de 253 
Hombre de 40 años. Previamente sano. PA: 3 años con disminución 
de  la  libido,  disminución  en  la  duración  de  las  erecciones  y  del 
volumen  de  semen.  Cefalea  frontal  y  biparietal  4/semana 
intensidad 8/10, que cede parcialmente con AINEs. Ha chocado 2 
veces  porque  “no  ve  al  auto  que  viene  al  lado”(sic).  
EF: TA 1120/80, FC 60xmin , FR 16. Peso 75kg , Talla 1.72. Cabeza, 
cuello  y  exploración  cardiopulmonar  sin  alteraciones.  Con 
ginecomastia, sin secreción por los pezones. Genitales Tanner IV, 
refiere  disminución  del  vello  corporal.  Exploración  neurológica: 
Campos  visuales  por  confrontación  con  disminución  del  campo 
temporal  bilateral. 
 
¿Cuál es la causa de hemianopsia bitemporal? 

 Hemorragia ocular 
 Compresión del quiasma óptico (10 Puntos) 
 Ninguna de las anteriores. 
 Infiltración del nervio óptico 
 Afección de la retina 

 
 
 

Pregunta 52 de 253 
¿De  acuerdo  a  la  sospecha  diagnóstica,  qué  estudios  deben 
realizarse inicialmente? 

 HLA‐B27, VSG, PCR, retinoscopía, OCT.  
 PRL,  PFT,  LH,  FSH,  Testosterona,  ACTH,  cortisol,  ES  (10 
Puntos) 
 PRL, PFT, LH, Testosterona  
 IRM de hipófisis. 

 
 
 

Pregunta 53 de 253 
Cuenta con los siguientes resultados: PRL: 20 μg/L. LH baja, FSH 
normal,  Testosterona  baja.  PFT  normales.  Cortisol  y  ACTH 
normales.  IRM  con  macroadenoma  de  22x31  mm  que  contacta  y 
desplaza  el  quiasma  óptico,  e  invade  seno  cavernoso  izquierdo. 
Campimetria  con  defecto  bitemporal  de  los  90  a  los  270º.  
 
¿A qué se debe que el nivel de prolactina sea normal? 

 Los hombres no producen prolactina.  
 El ensayo tiene poca sensibilidad 
 Efecto de gancho (10 Puntos) 
 Tiene un macroadenoma no productor. 
 Todas las anteriores. 

 
 
 

Pregunta 54 de 253 
En segunda determinación, se reporta prolactina dilución 1:100 250 
μg/L 
¿Cuál es el tratamiento de elección?  

 Bromocriptina + RTE 
 Cabergolina + radioterapia.  
 Cabergolina + RTE (10 Puntos) 
 Bromocriptina  
 RTE 

 
 
 

Pregunta 55 de 253 
¿ Cuál es la causa de los datos clínicos de hipogonadismo en este 
paciente? 

 Disminución de la producción de GH 
 Defecto congénito 
 Destrucción de las células gonadotropas 
 Disminución de producción de GnRH (10 Puntos) 
 Atrofia Testicular 

 
 
 

Pregunta 56 de 253 
Hombre de 45 años de edad. Campesino. PA: Cuadro de 6 meses 
con poliuria, polidipsia, polifagia, se realizó una glucosa capilar en 
ayuno  la  cual  fue  de  140.  Aparición  de  múltiples  verrugas  y 
pigmentación  negra  en  el  cuello.  Aumento  de  peso  de  8  kg  en  5 
años.  Su  ropa  le  sigue  quedando  pero  en  los  últimos  años  ha 
aumentado  la  talla  de  zapatos  de  6  a  7  ½.  
EF: TA 150/90, FC 90 x min, FR 18. Talla 1.72, peso 86kg. IMC 29.1 
Voz  ronca.Nariz  gruesa,  piel  grasa  algunos  comedones  en  nariz, 
prognatismo  y  engrosamiento  supraciliar.  Indentaciones  en  la 
lengua,  la  cual  es  geográfica.  Acantosis  nigricans  en  cuello  y 
pliegues.  Múltiples  acrocordones.  Manos  y  pies  anchos,  con 
abundante  tejido  adiposo  en  la  eminencia  tenar  y  los  talones.  
 
Principal causa de este síndrome clínico: 

 adenoma hipofisario productor de GH (10 Puntos) 
 craneofaringioma  
 idiopática  
 diabetes  
 secreción ectópica de GH 

 
 
 
Pregunta 57 de 253 
Mecanismo fisiopatológico de crecimiento acral: 

 aumento de todas las hormonas hipofisarias 
 proliferación anormal de vasos sanguíneos en respuesta a GH 
 efecto  directo  de  de  hormona  de  crecimiento  sobre  tejido 
conectivo.  
 aumento de IGF1 producida en hígado, estimulada por exceso 
de GH. (10 Puntos) 
 aumento de FGF23 

 
 
 

Pregunta 58 de 253 
Prueba de elección para el diagnóstico:  

 medición de IGF1 aislada 
 medición de IGF1 aislada y prueba de tolerancia a la glucosa de 
2hrs con medición de GH 
 medición de GH aislada 
 prueba de tolerancia a la glucosa de 2hrs con medición de GH 
 prueba de inhibición con 1mg de dexametasona nocturna.  

 
 
 

Pregunta 59 de 253 
Principal cáncer asociado a esta enfermedad endocrina: 
 cáncer de colon (10 Puntos) 
 cáncer de mandíbula 
 cáncer de estómago 
 cáncer de pulmón 
 no se asocia a ningún tipo de cáncer 

 
 
 

Pregunta 60 de 253 
Tratamiento de primera línea en esta patología: 

 radiación 
 cirugía (10 Puntos) 
 quimioterapia 
 cabergolina  
 vigilancia 

 
 
 

Pregunta 61 de 253 
Tratamiento médico recomendado: 

 cabergolina  
 radioterapia 
 bromocriptina  
 octreótide (10 Puntos) 
 No existe ningún tratamiento médico útil 
 
 
 

Pregunta 62 de 253 
Mujer  de  32  años,  residente  de  Chiapas.  Antecedente  de 
hipertensión arterial sistémica controlada con captopril. Acude por 
amenorrea  de  3  meses,  malestar  general  intenso,  fatiga, 
disminución en la concentración, intolerancia al frío, ortostatismo y 
síncope.  Los  síntomas  iniciaron  de  forma  lenta,  después  de  su 
último parto. G5, P4,C1, AO. Refiere haber tenido preeclamspia y 
abruptio placentae, requirió cesárea y transfusión de 3PG y 1 pool 
plaquetario.  Inicialmente  notó  imposibilidad  para  la  lactancia 
materna,  no  se  realizó  abordaje.  Ha  recibido  tratamiento  con 
multivitamínicos  sin  mejoría. 
EF:  TA  90/60  acostada,  80/50  de  pie.  FC  100x’  y  110x’  a  la 
bipedestación.Peso  70kg,  Talla  1.63.  Piel  y  mucosas  sin 
hiperpigmentación. Sin  bocio.  Disminución  de vello  axilar.  Taner 
mamario  4,  Taner  genital  III.  
Laboratorios: T3 0.4 ng/mL (0.64‐1.81), T4 3.5 ug/mL (5.91‐12.5), cT3 
28% (32‐48.8), TSH 0.5 (0.3‐5.0), Tg 11.82ng/dL (0‐36.8), Cortisol 5.4 
ug/dL (6.7‐22.6), ACTH 10 pg/ml (10‐100), Estradiol 2.5 g/mL (20‐
148),  FSH  0.1  mUI/mL  (3.0‐20),  LH  0.5mUI/mL  (2.0‐15). 
 
¿Cuál es el diagnóstico? 

 Insuficiencia suprarrenal primaria. 
 Deficiencia aislada del gonadotropo. 
 Panhipoituitarismo (10 Puntos) 
 Falla ovárica prematura. 
 Hipotiroidismo primario.  
 
 
 

Pregunta 63 de 253 
¿Cuál es la etiología en este caso? 

 Enfermedad infiltrativa 
 Craniofaringioma. 
 Síndrome de Sheehan . (10 Puntos) 
 Infecciosa. 
 Hipoxia neonatal. 

 
 
 

Pregunta 64 de 253 
¿Con qué hormona debe iniciarse el tratamiento de sustitución? 

 Hormona del crecimiento 
 Adrenalina 
 Hidrocortisona (10 Puntos) 
 Etinil‐estradiol 
 Levotiroxina 

 
 
 

Pregunta 65 de 253 
¿Cuál es un factor predisponente en esta paciente? 

 Lugar de residencia 
 Antecedente de hipertensión (10 Puntos) 
 Edad 
 Múltiples embarazos 
 Sobrepeso 

 
 
 

Pregunta 66 de 253 
Hombre  de  24  años.  Psicólogo.  AHF:  Abuelo  paterno:  vitiligo. 
Padre  y  hermana:  psoriasis.  Tíos  paternos  con  hipotiroidismo 
primario autoinmune. APNP: actividad física aeróbica y anaeróbica 
5  ves/semana,  consumo  de  suplementos  alimenticios.  COMBE 
negada. PA: hace 4 meses recibió tratamiento homeopático por acné 
(autotransfusión  intramuscular  glútea  de  6  ml  de  sangre)  y 
peróxido  de  benzoilo/clindamicina  tópico.  Tres  días  después 
presentó polidipsia con ingesta de 20L/d de agua fría, nicturia (x10), 
piel  seca,  intolerancia  al  ejercicio.  
EF:  Talla  1.77m,  Peso  83  kg,  IMC  26.49,  FC  50,  FR  16,  TA  110/70. 
Conformación  mesomórfica,  sin  facies  característica.  Bien 
hidratado. Cabeza  y  cuello  sin  alteraciones.  Campos  visuales  por 
confrontación  normales.  Cuello,  tórax,  abdomen  y  extremidades 
sin  alteraciones.  Tanner  V.  Exploración  neurológica  sin  datos 
anormales.  
Laboratorios: Hb 18.2g/dL. Glu 84mg/dL, Creatinina 1.2mg/dL, Na 
149 mEq/L, K 5 mEq/L, Cl 110 mEq/L, Ca 10.2 mEq/L, P 4.4 mEq/L, 
Alb  4.7  g/dL,  Osmolaridad  310  mOsm/mL,  NaU  28  mEq/L. 
Densidad  urinaria  1.007.  Proteinas  (‐).  pH  6.5,  NaU  28,  KU  18.6, 
CrU  19. 
El cuadro clínico corresponde a: 

 Síndrome de Sheehan 
 Diabetes mellitus 
 Diabetes insípida (10 Puntos) 
 Ninguno de los anteriores. 
 SIHAD 

 
 
 

Pregunta 67 de 253 
¿Qué estudio es útil para confirmar su sospecha? 

 Medición de ácido úrico 
 Prueba de deshidratación. (10 Puntos) 
 IRM de cráneo. 
 Curva de tolerancia a la glucosa oral. 
 USG renal  

 
 
 

Pregunta 68 de 253 
En la prueba previa, el paciente logró aumentar su OsmU 70% con 
respecto  a  la  basal,  con  administración  de  desmopresina.  El 
diagnóstico es:  

 Diabetes insípda central parcial 
 Diabetes insípda nefrogénica completa 
 Polidipsia primaria. 
 Diabetes insípda central completa (10 Puntos) 
 Diabetes insípda nefrogénica parcial 

 
 
 

Pregunta 69 de 253 
¿Cuál es la causa más frecuente de la enfermedad del paciente?: 

 Secundaria (10 Puntos) 
 Nefrogénica 
 Genética 
 Idiopática 
 Vascular 

 
 
 

Pregunta 70 de 253 
¿En este caso cuál es el tratamiento de elección? 

 Hidratación a libre demanda 
 Insulina 
 Restricción hídrica 
 Hidratación  a  libre  demanda  +  Desmopresina  IN  o  VO  (10 
Puntos) 
 Vasopresina IV. 
 
 
 

Pregunta 71 de 253 
Mujer  de  28  años.  Menarquia  12  años,  inicialmente  periodicidad 
regular.  G0,  P0.  Deseo  de  embarazo,  sin  MPF.  PA:  Inició  hace  6 
meses  con  irregularidad  menstrual,  ciclos  45‐65x5,  cefalea 
intermitente  biparietal  opresiva  5/10,  que  cede  con  paracetamol. 
Desde hace 2 meses presenta mastalgia y tumefacción mamaria. EF: 
TA 110/70, FC 80xmin , FR 19. Peso 62kg , Talla 1.55m. IMC 25.08. 
Cabeza,  cuello  y  exploración  cardiopulmonar  sin  alteraciones. 
Mamas simétricas sin nódulos. Secreción de líquido seroso‐blanco 
a  la  presión  de  ambos  pezones.  Sin  virilización  ni  datos  de 
resistencia  a  insulina.  Exploración  neurológica  normal.  Campos 
visuales  por  confrontación  normales.  
 
¿Cuál es la sospecha diagnóstica? 

 Falla ovárica prematura 
 Enfermedad de Cushing 
 Síndrome de Ovario poliquístico  
 Panhipopituitarismo  
 Prolactinoma. (10 Puntos) 

 
 
 

Pregunta 72 de 253 
¿Qué estudios requiere para confirmar el diagnóstico?  
 Medición de prolactina y PFT. (10 Puntos) 
 TAC de cráneo simple.  
 Prueba de estimulación con ACTH 
 PET‐CT  
 Ultrasonido transvaginal + medición de andrógenos. 

 
 
 

Pregunta 73 de 253 
La adenohipófisis produce las siguientes hormonas:  

 GH, oxitocina, ADH, FSH, Prolactina 
 GH, ACTH, prolactina, FSH, LH, FSH, ADH 
 GNRH, ACTH, ADH, TRH, oxitocina  
 GH, ACTH, prolactina, FSH, LH, TSH (10 Puntos) 

 
 
 

Pregunta 74 de 253 
El estudio imagenológico de elección en este caso es:  

 IRM de hipófisis con gadolinio. (10 Puntos) 
 Radiografía simple de cráneo 
 Octreoscan  
 TAC de cráneo contrastada 
 Mastografía 
 
 
 

Pregunta 75 de 253 
De  acuerdo  a  la  sospecha  clínica,  ¿cuál  es  el  tratamiento  de 
elección?  

 Somatostatina  
 Cabergolina (10 Puntos) 
 Cirugía transesfenoidal  
 Anticonceptivos orales combinados.  
 Radioterapia 

 
 
 

Pregunta 76 de 253 
Niño  de  12  años  con  obesidad  que  es  traído  por  su  madre  por 
presentar  cefalea  que  no  responde  a  analgésicos,  desde  hace  2 
semanas  bebe  y  orina  mucho  más  de  lo  normal.  
EF:  signos  vitales  normales,  sin  facies  característica,  sin 
deshidratación.  Obesidad,  resto  de  exploración  sin  alteraciones. 
Al  estudiarlo  la  diuresis  es  de  4  litros/día.  En  el  EGO  no  tiene 
glucosuria  ni  proteinuria.  La  osmolaridad  plasmática  es  de 
295mosm/L  y  la  osmolalidad  urinaria  es  de  100  mOsm/Kg.  
 
¿Cuál es el diagnóstico inicial? 

 Diabetes mellitus tipo 2 
 Diabetes mellitus tipo 1 
 Diabetes insípida (10 Puntos) 
 Síndrome de Prader Willi 
 Neuroblastoma. 

 
 
 

Pregunta 77 de 253 
Tras  someterlo  a  una  prueba  de  deshidratación,  la  osmolaridad 
urinaria es de 150 mOsm/Kg con una ADH plasmática por debajo 
de  las  cifras  normales.  Al  administrarle  vasopresina  SC,  la 
osmolalidad  urinaria  es  de  500  mOsm/Kg.  
 
De las siguientes la causa más probable de su patología es: 

 Síndrome de Wolfram 
 Hipopotasemia 
 Craneofaringioma (10 Puntos) 
 Porfiria aguda intermitente 
 Tuberculosis pulmonar 

 
 
 

Pregunta 78 de 253 
El estudio de gabinete que confirma el diagnóstico será: 

 Cateterismo de senos petrosos 
 Radiografía de cráneo en dos posiciones. 
 IRM de cráneo (10 Puntos) 
 PET‐CT 
 Electoencefalograma 

 
 
 

Pregunta 79 de 253 
El tratamiento de elección para este caso será 

 Octreótide 
 Cirugía conservadora + radioterapia local (10 Puntos) 
 Cirugía para resección extensa 
 Drenaje de quistes.  
 Quimioterapia 

 
 
 

Pregunta 80 de 253 
Una complicación frecuente de este tratamiento es:  

 Diabetes mellitus 
 Recidiva de la enfermedad 
 Pérdida del mecanismo normal de la sed 
 Panhipopituitarismo (10 Puntos) 
 Aplasia medular y alopecia 

 
 
 

Pregunta 81 de 253 
Mujer de 53 años. Obesidad mórbida, PO derivación gastroyeyunal 
laparoscópica hace 7 años. Presentó disminución de peso de 121.6 
Kg  a  68.5Kg. 
Tiene  diagnóstico  de  osteoporosis,  hipotiroidismo  primario, 
intolerancia  a  carbohidratos  y  enfermedad  ácido  péptica,  en 
tratamiento con En tratamiento con calcitriol, carbonato de calcio , 
levotiroxina  y  omeprazol  20.  La  paciente  acude  por  aumento  de 
peso de 5.8 kg en 2 años, con buen apego a alimentación y actividad 
física.  
EF: TA 140/90, FC76, FR 21. Peso 95, Talla 1.55, obesidad central, 
jiba  dorsal.  Sin  otros  datos  anormales  a  la  exporación.  Dentro  de 
estudio metabólico se encontró con glucemia en ayuno 125 mg/dL.  
Laboratorio:  cortisol  en  suero  matutino:  14.66  ug/dL,  cortisol  en 
suero  vespertino:  14.13  ug/dL.  Cortisol  en  orina:  485.2  ug/24h. 
ACTH:  3pg/mL.  Se  realizó  prueba  de  supresión  con  1  mg  de 
dexametasona:  Cortisol  AM  6.59  ug/dL. 
 
 
¿Cuál es el diagnóstico de la paciente?  

 Síndrome de Cushing por ACTH ectópica 
 Insuficiencia suprarrenal primaria 
 Hiperaldosteronismo primario 
 Enfermedad de Cushing 
 Pseudocushing 
 Síndrome de Cushing independiente de ACTH (10 Puntos) 

 
 
 

Pregunta 82 de 253 
¿Cuál es el siguiente estudio que debe solicitar?  

 Cateterismo de senos petrosos 
 Prueba de supresión con dexametasona 8 mg.  
 IRM de hipófisis 
 Prueba de supresión con dexametasona de 7 días con protocolo 
de Liddle 
 TAC de riñones y suprarrenales contrastada. (10 Puntos) 

 
 
 

Pregunta 83 de 253 
¿Cuál es el tratamiento de elección en este caso? 

 Ketoconazol  
 Resección transesfenoidal 
 Radioterapia 
 Metirapona  
 Adrenalectomía (10 Puntos) 

 
 
 

Pregunta 84 de 253 
En caso de que la opción de tratamiento no fuera plausible, usted 
indicaría  

 Ketoconazol VO (10 Puntos) 
 Dexametasona VO  
 Bloqueo alfa adrenérgico 
 Hidrocortisona VO 
 Octreótide IV 

 
 
 

Pregunta 85 de 253 
Es una complicación frecuente del tratamiento que seleccionó:  

 Hepatopatía (10 Puntos) 
 Síndrome de Cushing yatrógeno 
 Hipotensión ortostática 
 Diabetes mellitus 
 No tiene complicaciones 

 
 
 

Pregunta 86 de 253 
Hombre  de  52  años  de  edad,  campesino,  vive  en  hacinamiento. 
COMBE  positivo.  
Acude por fatiga crónica, debilidad muscular, sensación de mareo 
con  los  cambios  de  posición,  dolor  abdominal  intermitente,  en 
ocasiones asociado a vómito, y que desde hace algunos meses las 
encías  se  han  vuelto  moradas  y  las  manos  más  obscuras. 
Sudoración  nocturna  intermitente,  niega  tos  crónica.  
EF:  TA  100/60  acostado,  80/50  sentado  y  70/40  de  pie,  FC 
95acostado,  105  de  pie.  FR  18xmin.  Peso  65kg  Talla  1.70 
Hiperpigmentación de mucosas y pliegues, color bronce en toda la 
piel.  
 
¿Cuál es la sospecha diagnóstica? 

 Hipoaldosteronismo aislado 
 Cirrosis alcohólico‐nutricional  
 Cáncer suprarrenal 
 Insuficiencia suprarrenal primaria (10 Puntos) 
 Insuficiencia suprarrenal secundaria 

 
 
 

Pregunta 87 de 253 
.  El  mecanismo  fisiopatogénico  de  la  hiperpigmentación  en  este 
caso es: 

 Elevación de melanocortina  
 Por aumento de la proopiomelanocortina (POMC) (10 Puntos) 
 Aumentos de pigmentos circulantes  
 Por ACTH aumentada 

 
 
 
Pregunta 88 de 253 
Etiología más probable en este paciente: 

 Autoinmune 
 Cancer  
 Sarcoidosis  
 Linfoma 
 Tuberculosis (10 Puntos) 

 
 
 

Pregunta 89 de 253 
¿Qué estudio de gabinete es útil en este caso? 

 Octreoscan 
 Biopsia de ganglios  
 TAC de tórax 
 TAC de riñones y suprarrenales (10 Puntos) 
 IRM de silla turca 

 
 
 

Pregunta 90 de 253 
Es el fármaco de elección para el inicio del tratamiento: 

 Prednisona 
 Hidrocortisona (10 Puntos) 
 Fludrocortisona  
 Dexametasona 
 Metilprednisolona  

 
 
 

Pregunta 91 de 253 
Mujer de 32 años que acude por presentar 6 meses de evolución con 
aumento progresivo del peso, refiere haber subido 8kg, hirsutismo 
y ha notado aparición de estrías violáceas en abdomen, refiere caída 
de  cabello  y  debilidad  muscular.  
EF: TA 130/70, FC 95 lpm. Talla: 1.55 peso 90kg. Abdomen globoso, 
obesidad  ceentrípeta,  brazos  y  piernas  delgados,  con  estrías 
violáceas  en  abdomen,  brazos  y  piernas,  fragilidad  capilar 
generalizada.  
 
 
1.  ¿Cuál  es  la  alteración  metabólica  más  frecuente  en  esta 
enfermedad?  

 Hipoglicemia  
 Hipercalcemia 
 HDL bajas 
 Hipercolesterolemia 
 Hiperglucemia (10 Puntos) 

 
 
 
Pregunta 92 de 253 
¿Cuál es la principal alteración hidroelectrolítica en esta entidad?  

 Hipercalcemia 
 Hipofostatemia  
 Hiperkalemia  
 Hipocalcemia 
 Hipokalemia (10 Puntos) 

 
 
 

Pregunta 93 de 253 
La causa más frecuente de Síndrome de Cushing es:  

 Se desconoce  
 Hipofisiaria 
 Iatrogena (10 Puntos) 
 Suprarrenal 
 Ectopica  

 
 
 

Pregunta 94 de 253 
¿Qué estudio se requiere para confirmar la sospecha clínica?  

 Determinación de renina 
 Determinación de cortisol plasmático a las 8 AM  
 Determinación de ACTH  
 Determinación de acido vanilmandélico  
 Prueba de supresión con 1 mg de dexametasona (10 Puntos) 

 
 
 

Pregunta 95 de 253 
Es un fármaco que bloquea al receptor de corticosteroides:  

 Mifepristona (10 Puntos) 
 Lanreotide  
 Pasireotide  
 Ketoconazol  
 Metirapona  

 
 
 

Pregunta 96 de 253 
Paciente lactante menor, de 3 meses de edad, sexo legal masculino. 
Antecedente  de  un  hermano  de  6  años  con  pubertad  precoz. 
Embarazo y parto normal. Nació a las 39 SDG, peso al nacer de 3960 
gramos, talla nacimiento de 51 centímetros. Genitales externos con 
estructura  fálica  pequeña  con  hipospadias,  con  ausencia  de 
testículos en bolsa escrotal. Es traído por su madre por un cuadro 
de  3  días  de  evolución  caracterizado  por  compromiso  del  estado 
general,  vómitos  frecuentes  y  deshidratación  severa,  sin  otra 
sintomatología. Los exámenes de laboratorio evidenciaron Na 118 
mEq/L, K 6,1 mEq/L y gasometría venosa con acidosis metabólica 
descompensada,  sin  otra  alteración.  
 
¿Cuál es la sospecha diagnóstica? 

 Hiperplasia suprarrenal congénita clásica, variedad perdedora 
de sal (10 Puntos) 
 Síndrome de insensibilidad a andrógenos 
 Hiperplasia  suprarrenal  congénita  clásica,  variedad  virilizante 
simple 
 Exposición in útero a dietilestilbestrol 
 Hiperplasia suprarrenal congénita no clásica 

 
 
 

Pregunta 97 de 253 
El patrón de herencia de esta enfermedad es:  

 Autosómico dominante 
 Ligado al cromosoma X 
 Autosómico dominante con penetrancia incompleta 
 Autosómico recesivo (10 Puntos) 
 No es una enfermedad hereditaria 

 
 
 

Pregunta 98 de 253 
Cuál es el primer paso indicado en la atención de este paciente 
 Indicar corrección quirúrgica de los genitales ambiguos 
 Hidratación con solución salina y glucocorticoides (10 Puntos) 
 Iniciar taratamiento antibiótico para gastroenteritis 
 Iniciar protocolo de estudio de ambigüedad genital 
 Iniciar  infusión  de  bicarbonato  por  acidosis  metabólica 
decompensada 

 
 
 

Pregunta 99 de 253 
La enfermedad está causada por el siguiente defecto:  

 Mutación en el receptor de andrógenos 
 Deleción de un cromosoma X 
 Mutación en el gen de la 17alfa hidroxilasa 
 Mutación en el gen de la 11 beta hidroxilasa 
 Mutación en el gen de la 21 hidroxilasa (10 Puntos) 

 
 
 

Pregunta 100 de 253 
El diagnóstico se establece mediante 

 Medición de 17‐OH progesterona post estimulación con ACTH. 
(10 Puntos) 
 Cuadro clínico 
 Medición de tesosterona en sangre 
 Determinación del sexo cromosómico 
 Carga de sal 

 
 
 

Pregunta 101 de 253 
Hombre  de  35  años  de  edad,  con  diagnóstico  de  HAS.  En 
tratamiento con telmisartan 40mg/d, hidroclorotiazida 25 mg /d y 
posteriormente  amlodipino  20mg/d.  Acude  por  un  cuadro  de  5 
mese de evolución con debilidad proximal en miembros inferiores, 
dificultad  para  caminar  y  caídas.  
EF:  FC  89,  FR  20,  TA  160/100.  Fuerza  muscular  de  2/4  en  los 
miembros  inferiores  y  3/4  ensuperiores,  resto  de  exploración 
normal. Laboratorios: glucosa 94 mg/dl, creatinina 0.58 mg/dl, urea 
36 mg/dl, Na 148 mEq/l, K 2.7 mEq/l, Cl 100 mEq/l, P 3.1 mg, Ca 9 
mg/dl,  Mg  2.7  mg/dl. 
Gasometria  venosa  con  pH  7.47  y  HCO3  27.7  mEq/l.  
 
¿Cuál es la sospecha diagnóstica? 

 Síndrome de Cushing 
 Feocromocitoma 
 Estenosis de la arteria renal 
 Hiperaldosteronismo primario (10 Puntos) 
 Hipertensión esencial 

 
 
 
Pregunta 102 de 253 
El dato clínico más útil para establecer su sospecha es:  

 Hipokalemia (10 Puntos) 
 Hipertensión arterial 
 Tiempo de evolución 
 Función renal normal 
 Hipernatremia  

 
 
 

Pregunta 103 de 253 
La etiología más frecuente de hiperaldosteronsimo primario es:  

 Reninoma  
 Secreción ectópica de aldosterona 
 Carcinoma suprarrenal 
 Hiperplasia bilateral (10 Puntos) 
 Adenoma suprarrenal 

 
 
 

Pregunta 104 de 253 
El estudio de laboratorio útil para confirmar el diagnóstico es:  

 Prueba de supresión con 8 mg de dexametasona 
 Prueba de deshidratación.  
 Cortisol urinario de 24 horas 
 Prueba de hipoglucemia con medición de cortisol 
 PAC/PRA (10 Puntos) 

 
 
 

Pregunta 105 de 253 
Se  repuso  potasio  y  se  realizaron  los  siguientes  estudios:  cortisol 
12.49  μg/dl  (5‐25  μg/dl),  cortisol  urinario  70  μg/dl/24  h,  T4L  1.14 
ng/dl  (0.93‐1.70  ng/dl)  y  TSH  2.22  μU/l  (0.27‐4.2  μU/l).  
PAC 17.5 ng/dl y PRA 0.012 ng/ml/h, con relación PAC/PRA 1,458. 
Como prueba confirmatoria se realizo una carga de solucion salina, 
y  se  determino  PAC  14.8  ng/dl.  
TAC renal: glándula suprarrenal izquierda con lesión nodular, de 
8  ×  8  mm  con  comportamiento  radiológico  de  adenoma. 
 
La medida indicada en este caso es:  

 Cateterismo de venas suprarrenales 
 Adrenalectomía bilateral 
 Adrenalectomía izquierda (10 Puntos) 
 Espironolactona VO 
 Metirapona  

 
 
 

Pregunta 106 de 253 
Hombre de 44 años de edad, empresario. Tabaquismo activo IT 2. 
Antecedente de enfermedad ácido‐péptica tratada con pantoprazol 
40  mg/d,  dislipidemia  tratada  con  ezetimiba  y  simvastatina.  
Padecimiento  actual  de:  2  años  de  evolución,  caracterizado  por 
episodios de palpitaciones, cefalea holocraneana intensa, palidez y 
diaforesis.  Refiere  que  estos  síntomas  se  agravan  al  realizar 
actividad  física  (juega  tenis)  o  al  consumir  café  o  chocolate.  Ha 
notado  pérdida  de  7  kg  en  2  meses.  
EF: TA 168/110, FC 82 x‘, FR 22 x‘, peso 82 kg, talla 178 cm, IMC 26. 
Hidratado,  hiperemia  conjuntival  bilateral,  sin  alteraciones  a  la 
exploraci{on pulmonar, ápex desplazado a la izquierda. Abdomen 
plano  sin  tumores,  ni  visceromegalias,  se  palpa  pulso  aórtico  de 
intensidad  aumentada  sin  frémito  ni  soplo. 
Laboratorio:  BH.  Hb  14  Htc  32,  plaquetas  233,  leucocitos  8,600, 
neutrófilos 72%, linfocitos 19%, glucosa 102, BUN 23 Cr 1, Na 148, 
K  3.7,  Cl  107.  
 
El cuadro clínico es compatible con: 

 Feocromocitoma (10 Puntos) 
 Estenosis de la arteria renal 
 Hipertensión esencial 
 Hiperaldosteronismo primario 
 Crisis de angustia 

 
 
 

Pregunta 107 de 253 
Esta enfermedad puede relacionarse con una de las siguientes:  
 Li‐Fraumeni 
 Síndrome de Cowden 
 Von‐Hippel‐ Lindau (10 Puntos) 
 Síndrome de Marfán 
 Síndrome poliglandular autoinmune 

 
 
 

Pregunta 108 de 253 
De  los  siguientes,  el  método  con  mayor  sensibilidad  para  el 
diagnóstico es:  

 Metanefrinas libres en plasma (10 Puntos) 
 Tirosina en orina de 24 horas 
 Ácido vanilil‐mandélico 
 Cromogranina A 
 PAC/PRA 

 
 
 

Pregunta 109 de 253 
Al confirmar el diagnóstico del paciente. El tratamiento de elección 
es:  

 Cirugía (10 Puntos) 
 IECA 
 Ketoconazol 
 Antidepresivos 
 Espironolactona 

 
 
 

Pregunta 110 de 253 
Antes de llevar a cabo el procedimiento, es necesario indicar:  

 Mifepristona 
 Alfabloqueo seguido de betabloqueo adrenérgico (10 Puntos) 
 Beta bloqueo seguido de alfa bloqueo adrenérgico 
 No requiere preparación 
 Etomidato  

 
 
 

Pregunta 111 de 253 
Mujer  de  28  años.  Padece  vitiligo.  AHF:  una  hermana  con 
hipotiroidismo primario. Madre con vitiligo. Fumadora crónica, IT 
15.  
PA:  5  meses  de  evolución  con  ansiedad,  temblor,  “bochornos” 
hiperactividad, e insomnio. Ha perdido 10 kg en 2 meses, de forma 
involuntaria.  Tiene  mucha  hambre  e  hiperdefecación. 
Posteriormente  notó  crecimiento  del  cuello,  no  tiene  dolor  o 
síntomas  compresivos.  Tiene  dolor  ocular  leve  y  prurito.  Niega 
alteraciones  en  la  visión.  Algunos  familiares  le  han  hecho  notar 
cambios  en  sus  ojos.  
 
EF: TA: 130/80, FC 105lpm. FR 18rpm. Talla 1.56, peso 52kg, IMC 
21.36, peso previo de 63 kg, Exoftalmos bilateral, OD 23mm, OI 25 
mm. Eritema de la conjuntiva, retracción palpebral, sin restricción 
a los movimientos oculares, sin dolor espontáneo, sin diplopia y sin 
alteración a la visión de colores. Tiroides aumentada de tamaño 2 
veces  el  tamaño  normal,  blanda,  no  dolorosa.  No  se  palpan 
adenopatías. Temblor fino distal. Manos húmedas. REMs ¾ MS, 4/4 
MI.  
 
El cuadro clínico corresponde a: 

 Feocromocitoma 
 Cáncer de tiroides 
 Hipotiroidismo 
 Trastorno de ansiedad generalizada 
 Hipertiroidismo (10 Puntos) 

 
 
 

Pregunta 112 de 253 
Señala la afirmación incorrecta: 

 El defecto es una activación constitutiva del receptor de TSH (10 
Puntos) 
 El tratamiento de esta enfermedad incluye tionamidas. 
 La paciente tiene predisposición familiar 
 La etiología de este padecimeinto es autoinmune 
 Existen casos sin afectación ocular 

 
 
 

Pregunta 113 de 253 
Son estudios paraclínicos útiles para el diagnóstico:  

 Gammagrama tiroideo y anticuerpos TSI (10 Puntos) 
 Anticuerpos anti tiroglobulina  
 Anticuerpos anti hormonas tiroideas 
 Ultrasonido Tiroideo y BAAF 
 Cuantificación de TBG. 

 
 
 

Pregunta 114 de 253 
Se Realizan los siguientes estudios: T3 2.4 ng/mL (0.64‐1.81), T4 15.9 
ug/mL  (5.91‐12.5),  
cT3 52% (32‐48.8),TSH <0.03 (0.3‐5 ), Tg 39.82ng/dL (0‐36.8), Ac Anti 
TPO  Positivos,  Ac  Anti  Tg  
Positivos  ,  Ac  TSI  Positivos 
 
El diagnóstico final es:  

 Tiroiditis Subaguda de Quervain  
 Adenoma tóxico 
 Enfermedad de Graves 
 Carcinoma folicular de tiroides 
 Tirotoxicosis facticia 

 
 
 

Pregunta 115 de 253 
El tratamiento de elección es: 

 Resección de adenoma tóxico. 
 Suspensión de levotiroxina, evaluación psiquiátrica 
 Tiroidectomía total. 
 Metimazol, propranolol. (10 Puntos) 
 Prednisona, propranolol. 

 
 
 

Pregunta 116 de 253 
En caso de falta de respuesta o recaída está indicado: 

 Bolos de metilprednisolona 
 Administración de I131 (10 Puntos) 
 Tiroidectomía total 
 Ninguna opción es correcta 
 Radioterapia al cuello 

 
 
 

Pregunta 117 de 253 
Hombre  de  33  años  de  edad,  previamente  sano.  Sin  AHF 
relevantes. Acude a la consulta por un cuadro clínico de 12 meses 
de  evolución  caracterizado  por  aumento  progresivo  de  peso, 
dificultad  para  concentrarse,  astenia,  somnolencia  y  edema  de 
miembros  inferiores.  Ha  notado  leve  aumento  del  volumen  del 
cuello,  sin  síntomas  compresivos.  
 
EF: Talla 1,74 m, Peso 78 kg, IMC 25,72 kg/m2, TA 110/70, FC 65 xʹ.  
Paciente  bradipsíquico.  Piel  seca  y  desprendimiento  fácil  del 
cabello. Cabeza sin alteraciones. Ojos normales. A la palpación del 
cuello  se  encuentra  glándula  tiroides  aumentada  de  tamaño  de 
manera difusa, de consistencia firme. Exploración cardiopulmonar 
normal,  abdomen  sin  alteraciones.  Edema  sin  godete  en  ambos 
tobillo 
 
El cuadro clínico corresponde a: 

 Tiroiditis subaguda  
 Hipertiroidismo 
 Nódulo tiroideo  
 Hipotiroidismo (10 Puntos) 
 Cáncer de tiroides 

 
 
 

Pregunta 118 de 253 
La principal causa de bocio simple es:  

 Deficiencia de yodo (10 Puntos) 
 Deficiencia de TSH  
 Congénito  
 Cáncer de tiroides  
 Tiroiditis de Hashimoto 

 
 
 

Pregunta 119 de 253 
La causa de bradicardia en este padecimiento es:  

 Bloqueo AV de primer grado 
 Bloqueo de rama  
 Aumento de tono vagal  
 Hipertrofia cardíaca  
 Disminución en la respuesta a las catecolaminas. (10 Puntos) 

 
 
 

Pregunta 120 de 253 
¿Cuál de los siguientes es el perfil tiroideo esperado en este caso?  

 T3 normal, T4 normal, TSH alta  
 T3 baja, T4 baja, TSH baja  
 T3 alta, T4 alta, TSH alta.  
 T3 baja, T4 baja, TSH elevada (10 Puntos) 
 T3 alta, T4 alta, TSH baja 

 
 
 

Pregunta 121 de 253 
¿Cuál es la causa más frecuente de hipotiroidismo?  

 Hipotiroidismo secundario  
 Tumor hipofisiario que comprime al tirotropo.  
 Deficiencia de secreción de TRH 
 Hipotiroidismo primario (10 Puntos) 
 Yatrógeno.  

 
 
 

Pregunta 122 de 253 
El tratamiento de elección es:  

 Metimazol  
 Lugol  
 Levotiroxina (10 Puntos) 
 Propiltiuracilo  
 Triyodotironina  

 
 
 

Pregunta 123 de 253 
Hombre de 22 años de edad, es traído a la consulta por su mama, 
pues  ha  notado  que  esta  “hiperactivo”,  todo  el  tiempo  está 
despierto, ansioso, sudoroso, ha notado perdida de peso rápida, no 
cuantificada.  EF:  TA  130/70,  FC  95  lpm.  Talla:  1.75  peso  70kg. 
Tiroides ligeramente aumentada de tamaño, piel caliente, temblor 
distal,  diaforesis.  Dolor  retroocular,  edema  palpebral,  hiperemia 
conjuntival,  visión  normal.  
 
¿Cuál es la casusa más común de hipertiroidismo en el mundo?  

 TSHoma  
 Amiodarona.  
 Facticia  
 Cáncer folicular metastásico  
 Autoinmune (10 Puntos) 

 
 
 

Pregunta 124 de 253 
Mecanismo  fisiopatológico  de  la  oftalmopatía  en  enfermedad  de 
Graves  

 Acumulación  de  glucosaminoglicanos  de  fibroblastos  por 


estímulo de TSI (10 Puntos) 
 Hipertrofia de los músculos retrooculares por acción de T3.  
 Hipertrofia de la grasa retroocular en respuesta a TSH  
 Hipertensión ocular por exceso de catecolaminas.  

 
 
 
Pregunta 125 de 253 
Tratamiento de elección para la enfermedad de graves durante el 
embarazo:  

 Levotiroxina 
 Propanolol 
 Metimazol  
 Propiltiuracilo y propanolol (10 Puntos) 
 Vigilancia 

 
 
 

Pregunta 126 de 253 
Anticuerpos asociados a oftalmopatía distiroidea 

 TSI (10 Puntos) 
 Todos están asociados.  
 Anti TPO 
 Anti nucleares 
 Anti TSH 

 
 
 

Pregunta 127 de 253 
Mujer  de  61  años.  Empleada  un  laboratorio  de  radiodiagnóstico 
(toma las placas). Acude porque hace 3 meses notó la aparición de 
un nódulo en la cara anterior del cuello mientras deglutía. Niega 
síntomas compresivos, clínicamente eutiroidea. EF: TA 120/80, Fc 
80,  Fr  16,  Peso  60,  Talla  1,55.  Tiroides  aumentada  de  tamaño  a 
expensas  de  nódulo  tiroideo  izquierdo,  de  aproximadamente  1.5, 
blando  y  móvil.  No  se  palpan  adenopatías.  Resto  de  EF  normal 
 
¿Cuál es el riesgo de malignidad en este nódulo?  

 1% 
 10% 
 No tiene riesgo 
 5% (10 Puntos) 
 20% 

 
 
 

Pregunta 128 de 253 
¿Cuál es el dato que confiere riesgo de malignidad en este caso?  

 Ausencia de adenomegalias. 
 Trabajo con rayos X 
 Estar eutiroidea  
 Edad >60 años (10 Puntos) 
 Ser mujer 

 
 
 

Pregunta 129 de 253 
¿Qué estudios son útiles para el diagnóstico de forma inicial?  
 USG, TAC, biopsia excisional  
 TSH y USG (10 Puntos) 
 BAAF  
 TSH solamente 
 USG, TSH, Gammagrama.  

 
 
 

Pregunta 130 de 253 
Se  reporta  TSH  normal.  Nódulo  de  1.3  cm,  redondo,  con 
microcalcificaciones. ¿Cuál es el siguiente paso?  

 BAAF y gammagrama  
 I‐131  
 BAAF (10 Puntos) 
 Gamagrama tiroideo  
 Tiroidectomía total 

 
 
 

Pregunta 131 de 253 
El resultado de la citología es Bethesda I ¿cuál es el siguiente paso?  

 Vigilancia  
 Tiroidectomía  
 Biopsia excisional  
 TAC de cuello contrastada  
 Repetir BAAF en 3 meses. (10 Puntos) 

 
 
 

Pregunta 132 de 253 
Nueva citología con Bethesda II. ¿Cuál es el significado y el manejo 
recomendable?  

 Maligno. Tiroidectomía total.  
 Sospechoso de malignidad. Tiroidectomía total.  
 Benigno. Vigilancia (10 Puntos) 
 Atipia de significado incierto. Repetir en 3 meses  
 No diagnóstico. Repetir en 3 meses.  

 
 
 

Pregunta 133 de 253 
Mujer de 50 años, ama de casa. Previamente sana. Sin antecedentes 
familares importantes. Acude por aumento de volumen de la cara 
anterior del cuello, de 5 años de evolución, pero recientemente ha 
tenido crecimiento progresivo, asociado a disfagia alta a alimentos 
sólidos.  Sin  sintomas  de  distiroidismo.  
EF:  Peso  65  Kg,  Talla  1.60m,  FC  72,  FR  20,  TA  116/76.  
Tiroides aumentada al doble de su tamaño, de superficie irregular 
por  la  presencia  de  nódulo  en  lóbulo  izquierdo  de  2  cm  de 
diámetro,  de  consistencia  dura  y  móvil.  
En la cadena yugular izquierda se palpa una adenomegalia de 1 cm, 
fija,  blanda,  no  dolorosa.  
 
¿Qué  estudios  se  requieren  para  el  abordaje  inicial  de  la  lesión 
tiroidea? 

 Medición de tiroglobulina 
 Biopsia de tiroides a cielo abierto 
 Todas las opciones son correctas  
 TAC de cuello 
 Ultrasonido, TSH y BAAF (10 Puntos) 

 
 
 

Pregunta 134 de 253 
Tumor maligno más frecuente de tiroides: 

 Cáncer papilar (10 Puntos) 
 Cáncer medular 
 Metástasis. 
 Cáncer folicular 
 Cáncer anaplásico  

 
 
 

Pregunta 135 de 253 
La mutación asociada al carcinoma medular de tiroides es: 

 P53 
 Menina 
 Wolframina 
 RAS 
 RET (10 Puntos) 

 
 
 

Pregunta 136 de 253 
El  reporte  de  patología  fue:  Bethesda  IV,  carcinoma  papilar  de 
tiroides. El tratamiento indicado es: 

 Hemitiroidectomía izquierda 
 Tiroidectomía total (10 Puntos) 
 I 131 
 Radioterapia externa a cuello 
 I131 y quimioterapia  

 
 
 

Pregunta 137 de 253 
El seguimiento debe incluir:  

 No requiere seguimiento 
 USG de cuello cada 6‐12 meses (10 Puntos) 
 TAC de cuello anual 
 Octreoscan anual 
 Tiroglobulina cada mes 

 
 
 

Pregunta 138 de 253 
Mujer  de  20  años,  desempleada.  Antecedentes  familares:  ambos 
padres  diabéticos. 
Es  traída  por  su  hermana  porque  ha  notado  tinte  amarillo  en  su 
piel,  siempre  ha  tenido  “retraso  mental”.  Es  producto  del  tercer 
embarazo de madre de 29 años, nació a término, no recuerdan el 
Apgar.  A  los  5  meses  de  edad  su  madre  notó  detención  del 
crecimiento y falta de desarrollo neurológico (no se sentaba, nunca 
habló, casi no se movía). A los 8 años de edad acudió a un hospital, 
donde se inició tratamiento no especificado, con lo cual aumentó de 
estatura,  caminó  y  empezó  a  hablar,  sus  padres  suspendieron  el 
tratamiento.  Menarca  a  los  20  años,  ciclos  25  x  4.  Sin  actividad 
sexual.  
Refiere disnea de erfuerzo, tinte amarillo de la piel, intolerancia al 
frío,  estreñimiento,  piel  seca,  dificultad  para  hablar,  falta  de 
concentración, estreñimiento crónico, tristeza, labilidad emocional 
y  llanto  fácil.  
EF:  Talla  1.20,  Peso  45,  TA  110/70,  FC  65,  FR  19.  Edad  aparente 
menor  a  la  cronológica,  facies  cretinoide,  carotinemia,  piel  muy 
seca,  hiperpigmentación  de  labio  inferior,  no  se  palpa  tiroides, 
Tanner  mamario  II,  genital  II,  ausencia  de  vello  axilar. 
 
 
El cuadro clìnico corresponde a:  

 Hipotiroidismo congénito (10 Puntos) 
 Acondroplasia 
 Síndrome de Prader‐Willi 
 Enfermedad de Albright. 
 Hipoxia neonatal 

 
 
 

Pregunta 139 de 253 
La causa más común de este padecimiento es  

 Agenesia tiroidea familiar 
 Agenesia tiroidea esporádica (10 Puntos) 
 Exposición de la madre a tionamidas 
 Deficiencia de yodo 
 Exposición de la madre a radiación ionizante 

 
 
 

Pregunta 140 de 253 
El momento adecuado para la medición de TSH en los neonatos es  

 No se requiere medición en todos los neonatos 
 En las primeras 6 horas de vida 
 En las primeras 24 horas de vida 
 Al nacimiento 
 A las 48‐72 horas de vida (10 Puntos) 

 
 
 
Pregunta 141 de 253 
La aparición tardía de la pubertad en esta paciente:  

 Debe hacer pensar en una causa hereditaria 
 Es  una  manifestación  común  de  hipotiroidismo  congénito  (10 
Puntos) 
 Indica que el problema está a nivel hipofisario 
 Indica que la paciente es infértil.  
 Indica  falla  ovárica  prematura  y  obliga  a  descartar  síndrome 
poliglandular autoinmune  

 
 
 

Pregunta 142 de 253 
Por la hiperpigmentación en los labios, antes de iniciar tratamiento 
debemos descartar:  

 Melanoma 
 Insuficiencia suprarrenal secundaria 
 Complejo de Carney 
 Diabetes mellitus 
 Insuficiencia suprarrenal primaria (10 Puntos) 

 
 
 

Pregunta 143 de 253 
Hombre de 46 años, oficinista, tabaquismo desde hace 25 años, IT 
14. Antecedente familiar de infarto al miocardio en su madre a los 
60  años.  Acude  por  dolor  agudo  en  el  grupo  posterior  de  ambas 
piernas  al  caminar  del  bus  a  la  oficina,  que  se  alivia  con  reposo. 
EF:  peso  87  kg,  talla  168cm,  circunferencia  abdominal104cm, 
presión  arterial  140/80. 
Lab.  glucosa  140  mg/dl,  colesterol  total  240mg/dl,  Trigliceridos 
203mg/dl,  LDL  145  mg/dl,  HDLc  28  mg/dl. 
 
¿Cuál es la meta de HbA1c en este paciente? 

 9% 
 6.4% 
 Menos de 6% 
 Menos de 7% (10 Puntos) 
 8%  

 
 
 

Pregunta 144 de 253 
¿Cuál es la principal dislipidemia asociada con la obesidad? 

 Ninguna de las anteriores  
 Hipertrigliceridemia y HDL baja (10 Puntos) 
 Hipercolesterolemia 
 LDL baja 
 HDL altas 

 
 
 

Pregunta 145 de 253 
Principal hormona orexigénica  

 Catecolaminas  
 Insulina 
 Cortisol 
 Leptina  
 Ghrelina (10 Puntos) 

 
 
 

Pregunta 146 de 253 
Alteración pulmonar más frecuente en obesidad 

 EPOC 
 SAHOS (10 Puntos) 
 Bronquitis crónica 
 Cáncer  
 Asma  

 
 
 

Pregunta 147 de 253 
Dados  sus  factores  de  riesgo,  ¿qué  estudio  sería  recomendado 
realizar como escrutinio? 
 Electromiografía de miembros inferiores 
 Curva de tolerancia a la glucosa oral de 75 g (10 Puntos) 
 Urea marcada en aliento 
 Prueba de infusión con dexametasona 
 Coronariografía 

 
 
 

Pregunta 148 de 253 
Mujer de 54 años, sin diagnósticos previos, que acude a revisión en 
su empresa. Se detecta un índice de masa corporal de 32,8 kg/m2 y 
glucemia en ayunas 138 mg/l. Un mes después, glucemia 125 mg/dl. 
Está asintomática. A la EF TA 120/80, FC 80, FR 15, solamente llama 
la atención la presencia de obesidad y acantosis nigricans en cuello.  
 
¿Cuál es el diagnóstico de la paciente según sus cifras de glucosa? 

 No es posible establecer un diagnóstico. 
 Prediabetes (intolerancia a la glucosa) 
 Sana 
 Prediabetes (glucosa alterada en ayuno) (10 Puntos) 
 Diabetes mellitus tipo 2 

 
 
 

Pregunta 149 de 253 
¿Qué estudio está indicado realizar a continuación? 
 Prueba de supresión con 1 mg de dexametasona. 
 Pruebas de funcionamiento tiroideo 
 Prueba de ayuno. 
 Curva de tolerancia a la glucosa oral con carga de 100g.  
 Curva  de  tolerancia  a  la  glucosa  oral  con  carga  de  75g.  (10 
Puntos) 

 
 
 

Pregunta 150 de 253 
En  el  estudio  previo,  el  valor  registrado  a  las  2  horas  es  de 
190mg/dL  
¿Cuál es el diagnóstico? 

 Prediabetes (glucosa alterada en ayuno) 
 No es posible establecer un diagnóstico. 
 Sana 
 Diabetes mellitus tipo 2 
 Prediabetes (intolerancia a la glucosa) (10 Puntos) 

 
 
 

Pregunta 151 de 253 
¿Qué recomendación terapéutica efectuaría en primer lugar? 

 Insulina antes de cada comida. 
 Cambios conductuales: dieta y ejercicio físico. (10 Puntos) 
 Tomar acarbosa por la noche, antes de acostarse 
 Prescribir una sulfonilurea. 
 Administrar metformina. 

 
 
 

Pregunta 152 de 253 
Si  no  hubiera  resultados  adecuados  a  la  medida  anterior,  la 
indicación es: 

 No requiere más intervenciones 
 Iniciar insulina 
 Indicar sulfonilurea. 
 Enviar a cirugía bariátrica. 
 Indicar metformina. (10 Puntos) 

 
 
 

Pregunta 153 de 253 
Paciente  femenino  de  57  años  de  edad,  astenia,  adinamia, 
estreñimiento,  falta  de  concentración  y  ocasionalmente  debilidad 
proximal en extremidades inferiores. Cómo antecedente relevantes, 
litotripsia hace 3 años por cálculos renales. 

 El  paciente  no  tiene  alteraciones  del  metabolismo  del  calcio  y 


fósforo 
 Hipercalcemia 
 Síndrome de fatiga crónica 
 Hiperparatiroidismo primario (10 Puntos) 
 Mieloma múltiple 

Correcto 
 
 
 

Pregunta 154 de 253 
Que porcentaje del calcio plasmático se considera “activo” 

 35% 
 45% (10 Puntos) 
 25% 
 15% 
 50% 

COMENTARIO:  Dentro  del  plasma,  el  calcio  circula  unido  a  la 


albúmina en 40%, 15% forma complejos con citrato, fosfato y sulfato 
y 45% está en su forma ionizada o libre el cual se considera la forma 
activa 
 
 
 

Pregunta 155 de 253 
En base a su sospecha diagnóstica, usted solicita a su paciente: 

 Concentración sérica de hormona paratiroidea 
 Na, K y cloro 
 Calcio en orina de 24 horas 
 Ninguna de las anteriores. 
 A y C son correctas (10 Puntos) 

COMENTARIO:  El  diagnostico  de  hiperparatiroidismo  primario, 


en este caso, de paciente con hipercalcemia + osteoporosis, se puede 
confirmar  si  se  encuentran  concentraciones  de  PTH  altas  o 
inapropiadamente normales (ya que cuando existe un aumento del 
Ca, este es detectado por el sensor de Ca localizado en las glandúlas 
paratiroideas y la respuesta normal es disminución o supresión de 
la PTH). La supresión de PTH evitará sus efectos sobre el calcio, es 
decir que se absorba vía gastrointestinal (al inhibirse la conversión 
de 25‐di‐hidroxiVit D a 1‐25 hidroxi Vit D que esla responsable de 
laabsorción  intestinal  de  Ca)y  además  evitará  que  el  calcio  se 
reabsorba en el riñón. Por otro lado la determinación de Ca en orina 
de 24 horas ayudará al diagnóstico diferencial con Hipercalcemia 
hipocalciurica. 
 
 
 

Pregunta 156 de 253 
Una  vez  que  se  confirma  el  diagnóstico  de  Hiperparatiroidismo 
primario,  el  estudio  de  imagen  de  elección  para  determinar 
etiología y conducta terapeútica es: 

 Tomografia de cuello con foco en glándulas paratiroideas 
 Us de cuello de alta definición 
 Sestamibi con Tec 99 
 El sespaimi con tec 99 + Us de alta definicióne. Exploración de 
cuello vía laparoscópica (10 Puntos) 
COMENTARIO: El 85% del HPTP es por un adenoma único en una 
de  las  glándulas  paratiroideas,  el  gamagrama  con  captación  de 
tecnecio 99 (sestamibi) se considera el estudio de elección para la 
identificación  de  la  lesión  y  la  combinación  con  US  de  alta 
definición  de  cuello  mejora  la  probabilidad  de  encontrar  la 
glándula  enferma  y  definir  el  abordaje  quirúgico  de  mínima 
invasión,  la  sensibilidad  para  encontrar  el  adenoma  sube 
hastab95% (con setamibi solo es de 88%). 
 
 
 

Pregunta 157 de 253 
Las  siguientes  son  indicaciones  para  el  manejo  quirúgico  del 
paciente asintomático, excepto: 

 Menor de 55 años 
 Hipertensión arterial (10 Puntos) 
 Tasa de filtración glomerular menor de 60ml/mi 
 Calcio sérico arriba de 1mg/dL del l 
 Osteoporosis 

COMENTARIO:  Los  criterios  para  cirugía  de  HPTP  asintomático 


son : edad menor de 55, presencia de osteoporosis, TFG menor de 
60ml/hr,  fracturas  por  fragilidad  y  calcio  sérico  por  arriba  de 
1mg/dL  del  límite  normal.  La  hipertensión  no  es  un  criterio  de 
cirugía en el paciente asintomático ya que la corrección del HPTP 
no ha demostrado remitir la hipertensión. Una corta expectativa de 
vida es otra contraindicación para el paciente asintomático. 
 
 
 
Pregunta 158 de 253 
Paciente  masculino  de  30  años,  acude  a  consulta  por  obesidad 
desde  la  infancia.  Actualmente  con  IMC  34.  Refiere  depresión  y 
ansiedad. El paciente es sedentario. A la exploración física, usted 
encuentra:  Obesidad  generalizada,  estrías  abdominales  pálidas, 
giba dorsal, acantosis Nigricans y TA 145/95 mmhg. Su médico de 
primer  contacto  le  mandó  una  prueba  de  supresión  con  1mg  de 
dexametasoana, al dia siguiente de la ingestión de dexametasona 
su cortisol a las 8 am fue de 5 mg/dL. En base a lo siguiente, usted 
sospecha: 

 Obesidad mórbida 
 Síndrome de Cushing 
 Pseudo‐Cushing 
 Síndrome metabólico 
 Todas las anteriores, excepto d (10 Puntos) 

Correcto 
 
 
 

Pregunta 159 de 253 
Cómo siguiente exámen, usted solicita 

 COTG de 2 horas con 75 gr de glucosa 
 B y D son correctas (10 Puntos) 
 Concentraciones de ACTH 
 Cortisol salival vespertino 
 Cortisol en orina de 24 horas 
COMENTARIO: El paciente tiene todos los criterios de síndrome 
metabólico asociados a obesidad (la cual tmb, según los criterios de 
IDF  es  un  criterio  de  Sx  metabólico),  eso  aunado  a  depresión, 
propician un estado inflamatorio y de estrés que pueden originar 
hipercortisolismo  crónico  sostenido  y  un  cuadro  de  Pseudo‐
Cushing. El exámen físico NO apoya la presencia de Enfermedad 
de Cushing (ya que no muestra ningún dato “duro” del síndrome), 
esto  más  la  probabilidad  preprueba  (la  incidencia  de  Enf  de 
Cushing es de 1/100000, de predominio en mujer)NO justifican la 
búsqueda  de  este  raro  padecimiento.  El  abordaje  debe  ser 
enfocados a determinar su riesgo cardiometabolico global, debido 
a su obesidad mórbida,la COTG definirán si tiene diabetes, además 
es  importante  confirmar  que  tiene  “pseudocushing”,  páralo  cual 
está  indicado  medir  el  cortisol  salival  vespertino.  En  el  Pseudo‐
Cushing  se  pierde  el  ritmo  circadiano  de  la  secreción  de  cortisol, 
mismo que se conserva en la Enfermedad de Cushing, lo que ayuda 
a  una  fácil  discriminación  entre  ambos  padecimientos, 
particularmente en pacientes de muy baja probabilidad para tener 
Enf de Cushing. 
 
 
 

Pregunta 160 de 253 
El  patrón  de  dislipidemia  más  habitalque  que  se  encuentra  en 
pacientes con obesidad y síndrome metabólico es: 

 Hipertrigliceridemia severa (mayor de 1000mg/dL) 
 Hipercolesterolemia a expensas de C‐LDL alto 
 Hipertrigliceridemia leve 
 Colesterol HDL bajo, Trigliceridos altos (10 Puntos) 
 Ninguna de las anteriores 
COMENTARIO:  La  dislipidemia  clásica  de  los  pacientes  con  Sx 
metabólico  más  obesidad  es:  Hipertrigliceridemia  leve‐moderada 
con hipoalfalipoproteinemia (ambos son cirterios separados para sx 
metabólico, debido a su importancia). Parte de la fisiopatología de 
este patrón de dislipidemia reconocida como “aterogénica” se debe 
a  los  efectos  del  exceso  de  insulina  circulante,  con  aumento  de 
resistencia a su acción en tejidos perifericos como músculo y grasa 
visceral  con  sensibilidad  relativamente  conservada  a  nivel 
hepático. 
 
 
 

Pregunta 161 de 253 
El índice corporal del paciente indica que padece: 

 Niguna de las anteriores (10 Puntos) 
 sobrepeso más síndrome metabólico 
 Obesidad grado 2 
 Obesidad grado 3 
 Obesidad mórbida, ya que independientemente del IMC, tiene 
varios factores de riesgo cardiovascular 

COMENTARIO: Clasicamente, el IMC ha servido para diagnósticar 
sobrepeso  u  obesidad.  La  obesidad  grado  3,  cuando  el  índice  de 
masa  corporal  supera  40  kg/m2  es  llamada  obesidad  mórbida.  El 
IMC se calcula dividiendio el peso en kilos sobre la altura en mts al 
cuadrado. La clasificación es: 1‐ IMC 18.5‐24..9= Normal 2‐ IMC 25‐
29.9=sobrepeso 3‐ IMC 30‐34.9= obesidad grado 1 4‐ IMC 35‐39.9= 
Obesidad  grado  2  (está  obesidad  más  multiples  comorbilidades 
puede  ser  llamada  obesidad  mórbida)5‐  IMC  40  o  mayor= 
Obesidad grado 3= mórbida 
 
 
 

Pregunta 162 de 253 
Cómo primera medida terapeútica usted indica: 

 Por el momento indica estatinas, dieta y ejercicio y lo valorará 
en  un  mes  en  su  consulta  para  continuar  el  proceso  de 
empoderación. 
 Dieta muy baja en calorías para perder 10% del peso en 3 meses. 
 Solicita  IC  a  cirugía  ya  que  es  un  paciente  con  criterios 
quirúgicos,  que  si  se  opera  lo  antes  posible  se  puede  retrasar  el 
inicio de diabetes. 
 Abordaje  multidiciplinario  con  Nutrióloga, 
Psicócologo/psiquiatra  y  endocrinólogo  para  manejo  con  dieta 
terapeútica  con  reducción  de  500‐1000kg/dia,  manejo  médico  de 
depresión, dislipidemia, hipertensión y prediabetes (10 Puntos) 
 Es  prioritario  el  manejo  de  depresión  por  el  alto  riesgo  de 
suicidio 

COMENTARIO:  El  abordaje  de  este  caso,  debe  ser 


multidiciplinario,  con  una  estimación  de  su  riesgo 
cardiometabolico  global,  manejo  de  depresión  por  parte  de 
psicología/psiquiatría, manejo de las comorbilidades identificadas 
como  dislipidemi,  hipertensión  arterial  y  prediabetes.  Terapia 
nutricional,  con  diminución  de  600  Kcal  de  su  gasto  energético 
basal y restricción de azúcar refinada, carbohidratos (reducir a 45‐
50%  para  favorecer  la  disminución  de  TG  y  aumento  de  C‐HDL. 
Ejercicio  aeróbico  iniciar  con  150  min/semana  y  continuar  con 
300/min  semana  después  de  un  periodo  de  acondicionamiento 
físico  y  perdida  ponderal.  Empoderar  la  paciente  con  visitas 
continuas 
 
 
 

Cetoacidosis diabética 
Pregunta 163 de 253 
Hombre de 18 años, con diagnóstico de diabetes mellitus tipo 
1, se presenta a urgencias por un cuadro de 2 días con náusea, 
vómito, dolor abdominal y poliuria. EF: TA 120/60, FC 110x’, 
FR 30x’, respiración de Kussmaull, ligeramente deshidratado. 
Laboratorios:  glucosa  398  mg/dl,  Cr  1.0,  sodio  131  meq/L, 
potasio  4.8  meq/L,  gasometría  arterial  con  pH  de  7.2, 
bicarbonato 10 meq/L, CO2 24, SatO2 94. EGO: pH 6, leucos 
1/campo,  bacterias50/campo,  nitritos  (‐),  cetonas(+++). 
 
El diagnóstico clínico más probable es:  

 Hipoglucemia severa 
 Tormenta tiroidea 
 Estado hiperosmolar hiperglucémico 
 Cetoacidosis diabética (10 Puntos) 
 Intoxicación por salicilatos 

Respuesta  correcta:  cetoacidosis  diabética.  Se  trata  de  una 


complicación  de  DM  tipo  1,  la  más  frecuente  es  CAD,  el 
cuadro  clínico  es  compatible  al  ser  de  inicio  súbito, 
caracterizado  por  pérdida  de  peso,  poliuria,  polidipsia, 
deshidratación,  debilidad,  náuseas,  vómito  y  dolor 
abdominal.  Los  criterios  diagnósticos  para  CAD  son: 
Glucosa>250, pH <7.30, bicarbonato <15meq/L, presencia de 
cetonas  en  orina  y  anion  gap  >10.  Manual  de  terapéutica 
médica y procedimientos de urgencias INCMSNZ. Capitulo 
34. Cetoacidosis diabética. 
 
 
 

Pregunta 164 de 253 
El  tratamiento  inicial  de  esta  paciente  se  debe  efectuar 
administrando: 

 Bicarbonato de sodio IV  
 Solución glucosada al 50% 
 Insulina de acción rápida por vía intravenosa 
 Solución salina e insulina de acción rápida (10 Puntos) 
 Propranolol y metimazol 

Respuesta  correcta:  Solución  salina  e  insulina  de  acción 


rápida. El tratamiento se basa en la reposición de líquidos y 
electrolitos  y  la  administración  de  insulina  IV.  El  déficit  de 
agua en estos pacientes es de 5‐8 L, la reposición se inicia con 
solución salina 0.9% 1‐1.5L en la primera hora. Las siguientes 
soluciones  se  basan  en  el  estado  hídrico  y  el  sodio  del 
paciente. El uso de bicarbonato solamente está justificado en 
pacientes  con  pH<7.0.  El  tratamiento  con  insulina  debe 
iniciarse con un bolo de 0.1 UI/kg y posteriormente infusión 
a 0.1 UI/kg/h. Nunca debe iniciarse insulina si el paciente está 
inestable  o  el  potasio  es  bajo.  
Manual de terapéutica médica y procedimientos de urgencias 
INCMSNZ. Capitulo 34. Cetoacidosis diabética. 
 
 
 

Pregunta 165 de 253 
Para  definir  la  resolución  del  cuadro  se  requiere  la 
normalización de:  

 pH, cloro, diuresis, TA. 
 pH, bicarbonato, nivel de glucosa (10 Puntos) 
 pH, infecciones, hidratación 
 glucosa, insulina, soluciones 
 diuresis, estado de hidratación, bicarbonato 

Respuesta  correcta:  pH,  bicarbonato,  nivel  de  glucosa.  Los 


criterios  de  resolución  de  DM  son:  Glucosa<200,  pH  >7.35, 
bicarbonato  >18meq/L,  y  anion  gap  <10.  Manual  de 
terapéutica  médica  y  procedimientos  de  urgencias 
INCMSNZ. Capitulo 34. Cetoacidosis diabética. 
 
 
 

Pregunta 166 de 253 
Es  un  factor  desencadenante  de  esta  complicación, 
EXCEPTO:  

 Ejercicio extenuante (10 Puntos) 
 Pancreatitis 
 Omisión de insulina 
 Traumatismos 
 Infección 
Respuesta correcta: omisión de insulina. La CAD Es resultado 
de  la  disminución  absoluta  o  relativa  de  los  efectos  de 
insulina  y  del  incremento  de  las  hormonas 
contrarreguladoras.  Requiere  un  factor  desencadenante,  los 
más  importantes  son:  infección,  omisión  de  insulina,  estrés 
agudo(EVC,  IAM,  pancreatitis,  traumatismo,  quemaduras), 
embarazo y fármacos. Williams Textbook of endocrinology. 
Capitulo 33 Complications of Diabetes Mellitus.  
 
 
 

Pregunta 167 de 253 
Son hormonas de la respuesta contrarreguladora tardía, que 
están implicadas en la fisiopatología de esta complicación: 

 cortisol y GH (10 Puntos) 
 glucagón y adrenalina 
 glucagón e insulina 
 glucagón y GH.  
 ACTHy cortisol 

Respuesta  correcta:  cortisol  y  GH.  Williams  Textbook  of 


endocrinology.  Capitulo  33  Complications  of  Diabetes 
Mellitus.  
 
 
 

SIADH 
Pregunta 168 de 253 
Hombre de 50 años de edad, diabético, fumador crónico, IT 
40,  tos  crónica  con  expectoración  blanquecina.  Toma 
metformina 500mg cada 8hrs. Refiere que desde hace 2 meses 
ha presentado exacerbación de la tos, pérdida de peso de 15 
kg y astenia. Desde hace 24 horas presenta letargia, confusión 
y somnolencia. A la EF TA 110/60, FC 90lpm, bien hidratado, 
sin edema. Laboratorio: Glu 95, BUN 20, Cr 0.7, AU 3.2, Na 
125, K3.4, Cl 105, P3.2, osmolaridad urinaria 280 mOsm/kg, 
osmolaridad  plasmática  262mosm/kg.  
 
La osmolaridad sérica puede ser calculada si se conocen los 
valores séricos de sodio, potasio y: 

 glucosa y cloro. 
 bicarbonato y glucosa 
 bicarbonato y urea. 
 hematocrito y bicarbonato 
 nitrógeno uréico en sangre (bun) y glucosa. (10 Puntos) 

Respuesta  correcta:  Nitrógeno  uréico  en  sangre  (BUN)  y 


glucosa.  La  osmolaridad  sanguínea  es  un  parámetro 
importante  para  evaluar  el  equilibrio  hidrosalino.  Para 
calcularla usamos la fórmula: Osmp = (2 x Sodio)+ (Glucosa/ 
18)  +  BUN  /  2.8).  Williams  Textbook  of  endocrinology. 
Capitulo 10, Hipófisis posterior.  
 
 
 

Pregunta 169 de 253 
Receptor de vasopresina en túbulo contorneado distal: 
 alfa 
 V1 
 V2 (10 Puntos) 
 Beta 
 no tiene 

Respuesta correcta: V2. La ADH actúa en el receptor V2 en 
túbulo contorneado distal, y estimula la síntesis y migración 
celular de acuaporina‐2 hacia la membrana, que son canales 
a través de los cuales el agua se mueve a favor del gradiente 
osmótico  desde  el  conducto  colector  hacia  la  médula  renal. 
Williams Textbook of endocrinology. Capitulo 10, Hipófisis 
posterior.  

 
 
 

Pregunta 170 de 253 
Umbral osmótico para la secreción de ADH 

 300mosm 
 180mosm 
 320mosm 
 280mosm (10 Puntos) 
 270mosm 

La respuesta es la a, el umbral para que el osmostato capte 
una  disminución  en  el  volumen  circulante  es  apartir  de 
280mosm  y  empieza  la  secreción  de  vasopresina  para 
aumentar  el  liquido  intravascular. 
 
Respuesta  correcta:  280mosm.  La  secreción  de  vasopresina 
está  regulada  mediante  un  mecanismo  altamente  sensible, 
que  responde  a  cambios  mínimos  (1%)  en  la  osmolalidad 
plasmática  (umbral  280  mOsm/kg)  Williams  Textbook  of 
endocrinology. Capitulo 10, Hipófisis posterior.  

 
 
 

Pregunta 171 de 253 
En  el  síndrome  de  secreción  inapropiada  de  hormona 
antidiurética se produce: 

 hipernatremia – osmolaridad sérica normal 
 hipernatremia – hipoosmolaridad sérica 
 hiponatremia – hiposmolaridad sérica (10 Puntos) 
 hiponatremia – hiperosmolaridad sérica 
 hipernatremia – hipersosmolaridad sérica 

Respuesta correcta: hiponatremia – hiposmolaridad sérica. El 
SIADH  se  caracteriza  por  la  presencia  de:  Hiponatremia 
Euvolémica  +  Osmolalidad  plasmática  baja  (hipoosmolar)+ 
Osmolalidad  urinaria  elevada.  Los  criterios  de  diagnóstico 
son:  Na  <135,  OsmP  <275mOsm/kg.  Volumen  extracelular 
normal.  OsmU  >100mOsm/kg.  NaU  >40.  Ausencia  de 
hipovolemia,  ICC,  nefropatía,  hepatopatía  ISR  o 
hipotiroidismo.Sin  uso  reciente  de  diuréticos.  Williams 
Textbook of endocrinology. Capitulo 10, Hipófisis posterior.  
 
 
 
Pregunta 172 de 253 
el tratamiento del SIADH incluye 

 vasopresina 
 hidratación IV 
 diuréticos 
 tratar  la  causa  desencadenante  y  restricción  de  líquidos 
(10 Puntos) 
 Octreótide  

Respuesta  correcta:  tratar  la  causa  desencadenante  y 


restricción de líquidos. El tratamiento consiste en restricción 
hídrica y reposición de sodio. La corrección de Na debe ser 
cuidadosa. Se recomienda 1‐2 mEq/L/h las primeras 4 horas, 
luego  0.5mEq/l/h,  nunca  debe  excederse  de  10mEq/L  en  24 
horas.  Solo  el  tratamiento  de  la  causa  desencadenante  es 
efectivo  para  corregir  el  defecto.  Williams  Textbook  of 
endocrinology. Capitulo 10, Hipófisis posterior.  

 
 
 

HIPERTIROIDISMO 
Pregunta 173 de 253 
Hombre de 35 años, tabaquismo activo. Se presenta con un 
cuadro  de  3  meses  con  ansiedad,  diarrea,  temblor  de  las 
manos  y  pérdida  de  8kg. 
A  la  EF:  paciente  de  edad  aparente  a  la  cronológica,  TA 
160/100, FC 115 x min, FR 22, Temp 39.8ªC, peso 56kg, Talla 
1.61. la piel se ve lisa, caliente y suave, fácil desprendimiento 
del  cabello,  ojos  con  exoftalmos  leve,  retracción  palpebral, 
eritema  conjuntival  y  ardor  ocular,  sin  restricción,  visión 
normal.  Tiroides  aumentada  de  tamaño  2  veces  el  tamaño 
normal,  tiroides  blanda,  suave,  no  dorolosa.  Pemberton  ‐. 
Cardio pulmonar sin soplos, solo se ausculta ritmo cardiaco 
acelerado,  rítmico.  Temblor  distal  e  hiperreflexia. 
LABORATORIO:  leucocitos  10  800,  Neutrófilos  66.1  %,  Hb 
11.52 g/dL, Hcto 34.9%, Plaquetas: 187 000, glucosa 140, BUN 
12,  Creatinina  0.7,  bilirrubina  total  1.51,  BD  1.16,  BI  0.35, 
albumina 2.3, Na 139, Potasio 5.2, TGO 62, TGP 67. PFT: T3 
8.57  pg/ml,  T4  4.66  pg/ml,  TSH  <  0.005. 
 
El diagnóstico clínico más probable es: 

 tiroiditis de Quervain 
 cáncer del tiroides 
 bocio multinodular 
 bocio tóxico difuso (10 Puntos) 
 hipertiroidismo subclínico 

Respuesta correcta: Bocio toxico difuso. El bocio tóxico difuso 
(enfermedad  de  Graves)  es  la  causa  más  frecuente  de 
hipertiroidismo  en  el  adulto.  Combina  clínica  de 
hipertiroidismo (como la pérdida de peso o la alopecia), bocio 
difuso y signos extratiroideos como la oftalmopatía (hasta en 
la  mitad  de  los  pacientes)  o  la  dermopatía  (5‐10%  de  los 
casos).  La  ausencia  de  dolor  descarta  la  posibilidad  de 
tiroiditis de De Quervain. Greenspan, endocrinología básica 
y clínica, 9ª Ed. Capítulo 7. Glándula tiroides. 
 
 
 
Pregunta 174 de 253 
La  urgencia  cardiaca  más  frecuente  en  los  estados  de 
tirotoxicosis es:  

 fibrilación auricular con respuesta ventricular rápida (10 
Puntos) 
 taquicardia ventricular 
 fibrilacion ventricular 
 hipertensión arterial 
 insuficiencia cardiaca 

Respuesta  correcta:  fibrilación  auricular  con  respuesta 


ventricular  rápida.  La  principal  alteración  cardiaca  es 
fibrilación  auricular.  En  casos  de  tirotoxicosis  grave  puede 
presentarse  insuficiencia  cardíaca  congestiva.  Greenspan, 
endocrinología básica y clínica, 9ª Ed. Capítulo 7. Glándula 
tiroides.  
 
 
 

Pregunta 175 de 253 
El tratamiento de elección de la tiroiditis sub‐aguda es con: 

 yoduro reactivo. 
 metamizol. 
 propiltiuracilo. 
 propanolol y AIINES. (10 Puntos) 
 metimazol. 
Respuesta  correcta:  propanolol  y  AIINES.  La  tiroiditis 
subaguda  es  un  padecimiento  precedido  de  una  infección 
generalmente viral, cursan con una fase de hipertiroidismo, 
posteriormente  eutiroidismo  y  en  la  mayoría  de  los  casos 
generan  hipotiroidismo  finalmente.  El  tratamiento  consiste 
en el control de síntomas, con propanolol y algún análgesico 
no esteroideo. Greenspan, endocrinología básica y clínica, 9ª 
Ed. Capítulo 7. Glándula tiroides. 
 
 
 

Pregunta 176 de 253 
 
El tratamiento inicial del paciente debe ser con: 

 litio  
 alfa‐bloqueadores 
 preparaciones yodadas 
 levotiroxina 
 tionamidas (10 Puntos) 

 
Respuesta  correcta:  tionamidas.  No  existe  un  tratamiento 
específico contra el mecanismo patogénico en la enfermedad 
de  Graves.  El  tratamiento  se  basa  en  el  control  de 
hipertiroidismo con beta bloqueo, DAAT(tionamidas), I131 y 
en casos extremos con cirugía. En nuestro país el tratamiento 
más usado es DAAT y el fármaco de elección es metimazol 
10‐35mg/día VO. Greenspan, endocrinología básica y clínica, 
9ª Ed. Capítulo 7. Glándula tiroides.  
 
 
 

Pregunta 177 de 253 
En las mujeres embarazadas, el fármaco de elección para el 
tratamiento de hipertiroidismo es: 

 Propanolol 
 yodo radiactivo 
 Metamizol 
 propiltiouracilo (10 Puntos) 
 levotiroxina 

Respuesta  correcta:  propiltiuracilo.  No  está  disponible  en 


México, sin embargo de acuerdo a la bibliografía, se prefiere 
por  menor  incidencia  de  teratogenicidad  yporque  inhibe  a 
nivel periférico en la conversión de T4 a T3 y la desvía hacia 
T3r.  Greenspan,  endocrinología  básica  y  clínica,  9ª  Ed. 
Capítulo 7. Glándula tiroides.  
 
 
 

Pregunta 178 de 253 
El  efecto  adverso  más  grave  del  propiltiouracilo  es  la 
presentación de: 

 Hiponatremia 
 falla renal aguda 
 Hipoglucemia 
 agranulocitosis (10 Puntos) 
 pseudotumor cerebral 

Respuesta  correcta:  agranulocitosis.  Los  fármacos 


antitiroideos  o  tionamidas  (metamizol,  carbimazol  y 
propiltiouracilo)  constituyen  la  base  del  tratamiento 
antitiroideo.  la  reacción  más  severa  al  tratamiento 
antitiroideo es la agranulocitosis (definida como una cifra de 
granulocitos <500/mm3), que se presenta en uno de cada 500 
pacientes.  El  cuadro  es  de  aparición  brusca,  por  lo  que  la 
realización  de  recuentos  leucocitarios  frecuentes  no  es  útil 
para prevenir su aparición. Se presentan con fiebre y dolor de 
garganta. Greenspan, endocrinología básica y clínica, 9ª Ed. 
Capítulo 7. Glándula tiroides.  
 
 
 

COMPLICACIONES DE LA DM 
Pregunta 179 de 253 
Hombre de 68 años que padece diabetes mellitus 2 desde hace 
22 años, tratada con hipoglucemiantes orales, con mal control 
de la enfermedad. Desde hace diez años presenta distensión 
abdominal  postprandial  con  predomino  en  el  epigastrio,  la 
serie  esofagogastroduodenal  no  demuestra  alteraciones  y 
fluoroscopicamente se aprecia retraso en el vaciamiento del 
estomago. Además refiere la presencia de dolor ardoroso en 
plantas  y  palmas  y  visión  borrosa.  
 
La  aparición  de  las  complicaciones  microvasculares  y 
neurológicas de la diabetes a largo plazo se debe a: 
 acumulación de sorbitol (10 Puntos) 
 disminución de los receptores de la insulina 
 Glucotoxicidad 
 disminución de los factores de crecimiento semejantes a la 
insulina tipo II 
 edema vasogénico 

 
 
 

Pregunta 180 de 253 
De  las  siguientes,  la  manifestación  más  temprana  de  la 
nefropatía diabética en los estudios de laboratorio es: 

 la presencia de microalbuminuria. (10 Puntos) 
 el aumento progresivo de la creatinina sérica. 
 la presencia de cilindruria. 
 la disminución de la excreción de cistatina c. 
 la disminución del índice de filtración glomerular 

Respuesta correcta: La presencia de microalbuminuria . En la 
evolución  de  la  nefropatía  diabética  existen  los  siguientes 
estadíos:  
Estadío I: fase precoz no proteinúrica que se caracteriza por 
un  aumento  del  filtrado  glomerular.  El  aclaramiento  de 
Creatinina  y  el  transporte  máximo  de  glucosa  están 
aumentados. 
Estadío II: aparece eliminación intermitente de albúmina en 
orina,  fundamentalmente  en  relación  con  el  ejercicio.  
Estadío  III  o  nefropatía  incipiente:  se  caracteriza  por 
microalbuminuria  persistente  en  reposo.  Se  precisa  una 
determinación  de  microalbuminuria  .  La  microalbuminuria 
es  el  mejor  marcador  precoz  de  nefropatía. 
Estadío  IV  o  nefropatía  establecida:  caracterizada  por  una 
proteinuria  superior  a  los  300  mg/24h.  A  partir  de  este 
momento  se  produce  un  descenso  progresivo  de  la  tasa  de 
filtración  glomerular. 
Estadío  V:  se  caracteriza  por  insuficiencia  renal  grave.  En 
fases  finales  puede  existir  síndrome  nefrótico. 
Manual CTO 6ª Ed. Nefrología, Tema 9, Pág.31. 
 
 
 

Pregunta 181 de 253 
Los  pacientes  en  estado  prolongado  de  hiperglucemia 
pueden presentar catarata debido a: 

 la disminución de la glucosa en el cristalino 
 la formación y el acumulo de sorbitol en el cristalino. (10 
Puntos) 
 la oxidación de la glucosa en el cristalino. 
 la liberación de radicales libres en el cristalino. 
 el depósito de glucosa en el cristalino. 

Respuesta correcta: La formación y el acumulo de sorbitol en 
el cristalino . La catarata diabética se desarrolla debido a la 
transformación en el cristalino de glucosa a sorbitol, el cual 
no difunde a través de las membranas, acumulándose en el 
interior  del  mismo.  Greenspan,  endocrinología  básica  y 
clínica, 9ª Ed. Capítulo 17 Hormonas pancreáticas y diabetes 
mellitus 
 
 
 

Pregunta 182 de 253 
¿Cuándo se recomienda la primera revisión oftalmológica en 
diabetes tipo 2?  

 a los 10 años del diagnóstico  
 a los 5 años del diagnóstico  
 no se requiere revisión oftalmológica 
 al momento del diagnóstico (10 Puntos) 
 a los 3 años del diagnóstico 

Respuesta correcta: al momento del diagnóstico. La revisión 
de la retina debe realizarse en todos los pacientes con DM2 al 
momento  del  diagnóstico  y  en  los  DM1  a  los  5  años  del 
diagnóstico.  Standard  of  medical  care  in  diabetes.  Diabetes 
Care 2015; 38(1): 1‐99. Y Norma Oficial Mexicana NOM‐015‐
SSA2‐1994,  Para  la  prevención,  tratamiento  y  control  de  la 
diabetes. 
 
 
 

Pregunta 183 de 253 
La  principal  causa  de  muerte  en  pacientes  diabéticos  está 
relacionada con:  

 Eventos cardiovasculares (10 Puntos) 
 Hipoglucemias 
 cetoacidosis y estado hiperosmolar hiperglucémico 
 Neoplasias malignas 
 Infecciones 

Respuesta  correcta:  eventos  cardiovasculares.  Greenspan, 


endocrinología básica y clínica, 9ª Ed. Capítulo 17 Hormonas 
pancreáticas y diabetes mellitus 
 
 
 

CUSHING 
Pregunta 184 de 253 
Mujer de 35 años con obesidad grado III, hipertensión arterial 
y diabetes mellitus. Acude por cuadro de 1 años de evolución 
con fatiga fácil, debilidad muscular, amenorrea, hirsutismo, y 
aparición  de  estrías  abdominales  purpúreas.  
 
La causa más frecuente del síndrome de Cushing es:  

 administración de esteroides exógenos (10 Puntos) 
 adenoma de hipófisis secretor de ACTH  
 cáncer de células pequeñas de pulmón 
 tumor hipotalámico 
 adenoma suprarrenal  

Respuesta  correcta:  administración  de  esteroides  exógenos. 


La principal causa de sindrome de Cushing a nivel mundial 
es  la  Yatrógena  por  administración  de  esteroides.  De  los 
síndromes  de  hipercortisolismo  endógeno,  la  causa  más 
frecuente es Enfermedad de Cushing (adenoma hipofisiario 
productor de ACTH). Los adenomas suprarrenales le siguen 
en frecuencia. Williams Textbook of endocrinology. Capitulo 
5. La corteza adrenal.  
 
 
 

Pregunta 185 de 253 
La enfermedad de Cushing es secundaria a la: 

 terapéutica prolongada de esteroides sintéticos 
 hiperplasia suprarrenal. 
 secreción ectópica de hormona adrenocorticotrofica. 
 existencia de un adenoma hipofisario (10 Puntos) 
 existencia de un adenoma suprarrenal 

Respuesta correcta: existencia de un adenoma hipofisario. La 
enfermedad  de  Cushing  está  causada  por  un  tumor 
hipofisario  que  produce  grandes  cantidades  de  ACTH.  Los 
tumores son demostrables  en  el  80%  de  los  casos. Williams 
Textbook of endocrinology. Capitulo 5. La corteza adrenal.  
 
 
 

Pregunta 186 de 253 
Como  parte  de  la  respuesta  metabolica  al  trauma,  los 
glucocorticoides:  

 estimulan  la  liberación  de aminoácidos  desde  los tejidos 


periféricos (10 Puntos) 
 incrementan el calcio citosólico  
 estimulan  los  alfarreceptores  y  disminuyen  el  calcio 
citosólico  
 inhibe la salida de aminoácidos del musculo  
 disminuyen  la  liberación  de  glicerol  y  estimulan  a  los 
alfarreceptores  

Respuesta  correcta:  estimulan  la  liberación  de  aminoácidos 


desde los tejidos periféricos. El efecto de las glucocorticoides 
influye  en  el  metabolismo  de  las  proteínas  generando 
catabolismo, de los líquidos estimulando la lipolisis, y de los 
hidratos  de  carbono  inhibiendo  la  liberación  de  insulina  y 
aumentando  las  concentraciones  de  glucosa  a  partir  de  la 
gluconeogénesis  hepática.  Williams  Textbook  of 
endocrinology. Capitulo 5. La corteza adrenal.  
 
 
 

Pregunta 187 de 253 
Es  un  fármaco  que  inhibe  la  producción  de  cortisol  a  nivel 
suprarrenal:  

 Ketoconazol (10 Puntos) 
 Mifepristona 
 Pasireotide 
 Lanreotide  
 Metirapona  

Respuesta  correcta:  Ketoconazol  .Algunos  pacientes 


requieren  tratamiento  de  bloqueo  adrenal  previo  al 
tratamiento quirúrgico o no pueden operarse, se utilizan los 
siguientes  fármacos:  
Metirapona:  Bloquea  11β‐OHasa,  disminuye  cortisol.  No 
disponible  en  México 
Ketoconazol:  Inhibe  producción  cortisol  por  11β‐Ohasa,  17‐
OHasa,  18‐Ohasa.  Dosis  terapéutica  400‐1200  mg./d. 
Hepatotoxicidad.  Eficacia  de  70%.  
Mifepristona:  antagonista  receptor  glucocorticoides. 
Actividad  anti‐progestina  y  débil  anti‐andrógenos.  Eficacia 
50‐85% 
Pasireotide:  ligando  del  receptor  de  somatostatina 
multireceptor.  600μg  2  veces  día/15  días.  Eficacia  76‐87%  
Williams Textbook of endocrinology. Capitulo 5. La corteza 
adrenal.  
 
 
 

Pregunta 188 de 253 
Es una complicación frecuente del fármaco que seleccionó:  

 hiperplasia suprarrenal 
 diabetes mellitus 
 aborto espontáneo 
 hepatopatía (10 Puntos) 
 apoplejía hipofisaria.  

Respuesta correcta: hepatopatía. La hepatopatía es el efecto 
adverso  más  común  de  ketoconazol.  Está  indicada  la 
suspensión  del  mismo  cuando  las  transaminasas  se  elevan 
por  5  a  10  veces  el  valor  normal  durante  el  tatamiento. 
Williams Textbook of endocrinology. Capitulo 5. La corteza 
adrenal. 
 
 
 

INSUFICIENCIA SUPRARRENAL 
Pregunta 189 de 253 
A 43‐year‐old man with a history of active smoking, chronic 
hypertension and type 2 diabetes mellitus treated with oral 
hypoglycemic  agents,  presents  at  emergency  room  with 
fatigue,  weakness,  weight  loss,  hyperpigmentation  of  skin 
and  mucous  membranes,  symptomatic  hypoglycemia  and 
diffuse abdominal pain, nausea, and vomiting, since 2 weeks 
ago.  
Physical exam: BP 70/60, CR 110, 19 RR 29, lungs with bibasal 
crackles.  Abdomen  soft  without  peritoneal  irritation. 
Laboratories: Glucose 58 19 BUN, Cr 1.1, sodium 123 mEq / l, 
5.3  meq  potassium,  12  g  Hb  /  dL  Leukocytes  19mil. 
 
To  make  the  diagnosis  of  chronic  primary  adrenal 
insufficiency must be found: 

 hypokalemia, hyponatremia and orthostatic hypotension 
 hyperpigmentation,  orthostatic  hypotension  and 
hyperkalemia. (10 Puntos) 
 hyperpigmentation, hyperglycemia and hypokalemia. 
 weakness, arterial hypertension and hypokalemia. 
 osteoporosis, hypertension and hyperglycemia. 

Correct answer: hyperpigmentation, orthostatic hypotension 
and  hyperkalemia.The  most  common  symptoms  of  adrenal 
insufficiency  include  progressive  weakness  and  fatigue, 
hypoglycemia,  weight  loss  and  gastrointestinal  symptoms 
(abdominal pain, nausea and vomiting). The primary disease 
includes  involvement  mineralocorticoid  secretory  tissue 
causing hipoaldosteronism with sodium loss, hyperkalemia, 
avidity for salt, orthostatic hypotension and mild metabolic 
acidosis. Lack of cortisol produces a compensating increase 
in  ACTH  synthesis,  leading  to  the  characteristic  skin 
hyperpigmentation.  Williams  Textbook  of  Endocrinology. 
Chapter 5. The adrenal cortex. 

 
 
 

Pregunta 190 de 253 
The main cause of adrenal insufficiency is 

 trauma 
 bilateral adrenal infarction. 
 tuberculosis 
 anticoagulation 
 autoimmune disease (10 Puntos) 

Adrenal insufficiency is a result of failure of the adrenal gland 
(primary),  deficient  secretion  of  ACTH  by  the  pituitary 
(secondary)  or  CRH  from  the  hypothalamus  (tertiary).  The 
main  cause  of  primary  adrenal  insufficiency  in  developed 
countries  is  the  autoimmune  adrenalitis,  while  the  primary 
cause  of  failure  is  high  surprarrenal  abrupt  withdrawal  of 
exogenous  glucocorticoids.  Williams  Textbook  of 
Endocrinology. Chapter 5. The adrenal cortex. 
 
 
 

Pregunta 191 de 253 
The steroid of choice for treating adrenal crisis is: 

 betamethasone 
 methylprednisolone 
 dexamethasone 
 prednisolone 
 hydrocortisone (10 Puntos) 

Correct  answer:  hydrocortisone.  The  substitution  treatment 


of Addisonʹs disease requires steroids with medium or short 
half‐life,  such  as  hydrocortisone  or  prednisone. 
Hydrocortisone  possesses  mineralocorticoid  action,  so  is 
considered  the  steroid  of  choice,  plus  adequate  water 
replacement.  Williams  Textbook  of  Endocrinology.  Chapter 
5. The adrenal cortex. 
 
 
 

Pregunta 192 de 253 
A  drug  that  can  trigger  adrenal  crisis  in  patients  with 
untreated adrenal insufficiency. 

 aspirin 
 lithium 
 hydrochlorothiazide 
 amiodarone 
 levothyroxine (10 Puntos) 
Correct answer: levothyroxine. In patients with concomitant 
hypothyroidism  and  adrenal  deficiency,  the  steroid 
replacement must be the first step, since the administration of 
thyroid hormones may precipitate an adrenal crisis. Williams 
Textbook of Endocrinology. Chapter 5. The adrenal cortex. 
 
 
 

Pregunta 193 de 253 
The effect of aldosterone in the kidney is: 

 inhibits the basolateral Na+ / K+ pump 
 increase  the  number  of  aquaporins  in  plasmatic 
membrane 
 stimulates the basolateral Na+ / K+ pump (10 Puntos) 
 opens the voltage‐dependent calcium channels  
 inhibits the conversion of angiotensinogen to angiotensin 

Correct  answer:  stimulates  the  basolateral  Na+  /  K+  pump. 


Aldosterone acts on the renal tubular sodium transport in the 
distal convoluted tubule, by stimulating the basolateral Na+/ 
K+ pump, increasing the reabsorption of sodium and water; 
and  increasing  potassium  excretion.  Also  converts 
angiotensinogen  to  angiotensin  I.  Williams  Textbook  of 
Endocrinology. Chapter 5. The adrenal cortex. 
 
 
 

PROLACTINOMA 
Pregunta 194 de 253 
A  42‐year‐old  obese  woman  with  type  2  diabetes,  diabetic 
retinopathy,  hyper¬tension,  and  hirsutism  presents  to 
discuss  an  elevated  prolactin  level  of  144.8  ng/mL  (normal 
range, 4.8 to 23.3 ng/mL) found by her Ob‐Gyn two months 
ago. She complained of galactorrhea and no menses for one 
year. A repeat prolactin level was also elevated, at 109 ng/mL. 
 
The following are causes of hyperprolactinemia, EXCEPT: 

 chronic renal or hepatic failure 
 pregnancy 
 obesity (10 Puntos) 
 estrogen use 
 hypothyroidism 

Correct  answer:  obesity.  There  are  many  causes  of 


hyperprolactinemia.  Factors  that  can  increase  prolactin 
secretion  include  pregnancy,  nursing,  physiologic  stress, 
estrogen  use,  polycystic  ovary  syndrome,  hypothyroidism, 
and  chronic  renal  or  hepatic  failure.  Head  trauma,  use  of 
certain medications (verapamil, neuroleptics, antipsychotics, 
and antidepressants), and presence of nonsecretory sellar or 
suprasellar  masses  can  also  increase  prolactin  levels.  In 
general,  signs  and  symptoms  are  due  to  either  the  effect  of 
excess hormone secretion (ie, galactorrhea and amenorrhea) 
or  local  compression  (ie,  new‐onset  or  persistent  headache, 
dizziness,  visual  changes,  and  vision  loss).  A  review  of 
medications,  including  estrogen  therapy,  and  history  of 
fertility  or  gonadal  dysfunction  should  be  documented. 
Elevated  prolactin  levels  can  result  in  secondary 
hypogonadism.  Williams  Textbook  of  Endocrinology. 
Chapter 9.Pituitary masses and tumors. 
 
 
 

Pregunta 195 de 253 
The size of microadenomas does not exceed:  

 10mm (10 Puntos) 
 2mm 
 4mm 
 6mm 
 8mm 

Correct  answer:  10mm.  A  microadenoma  is  defined  as  an 


intrasellar  mass  less  than  10  mm  in  diameter.  A 
macroadenoma,  defined  as  larger  than  10  mm  in  diameter, 
can cause enlargement of the sella turcica. Williams Textbook 
of Endocrinology. Chapter 9.Pituitary masses and tumors. 
 
 
 

Pregunta 196 de 253 
The drug of choice for treatment of prolactinoma is: 

 danazol 
 tamoxifen 
 leuprolide acetate 
 cabergoline (10 Puntos) 
 mestranol 

Correct answer: cabergoline. Optimal treatment outcomes for 
a  prolactinoma  include  normalization  of  PRL  levels  (and 
associated  signs  and  symptoms)  and  complete  tumor 
removal or shrinkage with a reversal of tumor‐mass effects. 
Medical  management  of  prolactinomas  with  dopamine 
agonist  drugs  has  been  widely  recommended  as  the 
treatment  of  choice.  Bromocriptine  and  cabergoline  lowers 
elevated PRL levels, restores abnormal menstrual function in 
80%  to  90%  of  patients,  shrinks  prolactinomas,  restores 
impaired  sexual  function,  and  improves  galactorrea. 
Cabergoline  has  a  longer  duration  of  action  than 
bromocriptine  and  is  usually  administered  once  or  twice 
weekly. Since its introduction, it has surpassed bromocriptine 
as the first‐line therapeutic choice for most patients. Williams 
Textbook of Endocrinology. Chapter 9.Pituitary masses and 
tumors. 
 
 
 

Pregunta 197 de 253 
The mechanism of action of the drug you chose is: 

 anti‐androgenic 
 GnRH analogues  
 dopaminergic agonists (10 Puntos) 
 dopaminergic antagonists 

Correct answer: dopaminergic agonists. Medication therapy 
involves treatment with a dopamine agonist, which directly 
inhibits  prolactin  secretion  by  the  tumor  and  therefore 
suppresses tumor growth. The goal of medication therapy is 
to  suppress  the  prolactin  level  to  normal  range  and  restore 
gonadal  function.  The  two  dopamine  agonists  used  are 
bromocriptine  and  cabergoline.  Williams  Textbook  of 
Endocrinology. Chapter 9.Pituitary masses and tumors. 
 
 
 

Pregunta 198 de 253 
What  is  the  cause  of  hypogonadism  in  patients  with 
prolactinoma? 

 testicular atrophy 
 decreased production of GnRH (10 Puntos) 
 destruction of gonadotropic cells 
 congenital defect 
 decreased production of GH 

Correct  answer:  decreased  production  of  GnRH.  Elevated 


PRL  causes  sexual  dysfunction  via  a  short‐loop  feedback 
effect  on  gonadotropin  pulsatility  that  presumably  inhibits 
GnRH  as  well  as  LH  pulse  frequency  and  amplitude.  High 
PRL  also  directly  inhibits  ovarian  and  testicular  function. 
Williams  Textbook  of  Endocrinology.  Chapter  9.Pituitary 
masses and tumors. 

 
 
 
FEOCROMOCITOMA 
Pregunta 199 de 253 
A 43‐year‐old woman with hypertension, latent autoimmune 
diabetes  of  adulthood  and  autoimmune  hypothyroidism 
presented at emergency room with a three‐month history of 
altered glicemic control and uncontrolled hypertension with 
concomitant  recurrent  urinary  tract  infections.  Two  weeks 
ago  she  had  paroxysms  of  anxiety,  palpitations  and 
diaphoresis.  
 
¿What is the most likely diagnosis?: 

 essential hypertension 
 Primary hyperaldosteronism 
 Pheochromocytoma (10 Puntos) 
 generalized anxiety disorder 
 Renal artery stenosis  

Correct  answer:  Pheochromocytoma.  Pheochromocytoma 


has  a  variable  clinical  presentation.  It  often  appears  as 
paroxysms <1 hour, palpitations, headache, profuse sweating 
(classic triad), hypertension, anxiety and pale skin. Williams 
Textbook  of  endocrinology.  Chapter  16.  Endocrine 
hypertension.  
 
 
 

Pregunta 200 de 253 
One of the following can be related to this disease:  
 Marfan Syndrome 
 Von‐Hippel‐ Lindau Disease (10 Puntos) 
 Li‐Fraumeni Syndrome 
 Autoimmune polyglandular syndrome 
 Cowden Syndrome 

Correct  answer:  Von‐Hippel‐  Lindau  Disease. 


Catecholamine‐secreting tumors occur with equal frequency 
in men and women, primarily in the third, fourth, and fifth 
decades.  These  tumors  are  rare  in  children,  and  when 
discovered,  they  may  be  multifocal  and  associated  with  a 
hereditary síndrome as Von‐Hippel‐ Lindau Disease, MEN 1, 
MEN2A,  MEN2B,  or  neurofibromatosis  type  1.  Williams 
Textbook  of  endocrinology.  Chapter  16.  Endocrine 
hypertension.  
 
 
 

Pregunta 201 de 253 
What is the mutation present in multiple endocrine neoplasia 
type 2A? 

 wolframin 
 RAS 
 p53 
 RET. (10 Puntos) 
 menina 

Correct  answer:  RET.  The  MEN  type  2  is  caused  bya 


mutations in the proto‐oncogene RET, and is manifested by 
the  presence  of  medullary  thyroid  cancer,  and  primary 
hyperparathyroidism.  Williams  Textbook  of  endocrinology. 
Chapter 16. Endocrine hypertension.  
 
 
 

Pregunta 202 de 253 
The treatment of choice for this disease is: 

 spironolactone 
 antidepressants 
 surgery (10 Puntos) 
 IECA 
 ketoconazole 

Correct  answer:  surgery.  The  treatment  of  choice  for 


pheochromocytoma  is  complete  surgical  resection.  Surgical 
survival rates are 98% to 100% and are highly dependent on 
the  skill  of  the  endocrinologist,  endocrine  surgeon,  and 
anesthesiologist team. Williams Textbook of endocrinology. 
Chapter 16. Endocrine hypertension.  
 
 
 

Pregunta 203 de 253 
Preoperative management should include: 

 beta‐adrenergic  blockade  followed  by  alpha‐adrenergic 


blockade 
 alpha  adrenergic  blockade  followed  by  beta‐adrenergic 
blockade (10 Puntos) 
 no preparation is required. 
 mifepristone 
 etomidate 

Correct answer: alpha adrenergic blockade followed by beta‐
adrenergic  blockade.  Some  form  of  preoperative 
pharmacologic preparation is indicated for all patients with 
catecholamine‐secreting  neoplasms.  Combined  beta  and 
alpha adrenergic blockade is one approach to control blood 
pressure  and  prevent  intraoperative  hypertensive  crises. 
Alpha‐adrenergic  blockade  should  be  started  7  to  10  days 
preoperatively to normalize blood pressure and expand the 
contracted  blood  volume.  After  adequate  alpha  adrenergic 
blockade  has  been  achieved,  beta‐adrenergic  blockade  is 
initiated,  typically  2  to  3  days  preoperatively.  Williams 
Textbook  of  endocrinology.  Chapter  16.  Endocrine 
hypertension 
 
 
 

CÁNCER DE TIROIDES 
Pregunta 204 de 253 
A  34‐year‐old  woman  presented  with  an  asymptomatic 
enlarging nodule of the right thyroid lobe. Ultrasonography 
revealed  a  diffusely  enlarged  right  thyroid  lobe  with  three 
hypoechogenic lesions, the largest of which measured 5 cm 
in maximum dimension. Fine Needle Aspiration biopsy was 
performed,  which  showed  follicular  epithelial  cells,  with 
optically  clear  nuclei  and  nuclear  grooves,  suggestive  of  a 
diagnosis  of  Papillary  thyroid  carcinoma.  Total 
thyroidectomy  was  performed.  
 
What is the risk of malignancy in thyroid nodules? 

 No risk 
 20% 
 10% 
 5% (10 Puntos) 
 1% 

Correct answer: 5%. A thyroid nodule is a lesion within the 
thyroid  gland  that  is  distinguished  from  the  surrounding 
thyroid  parenchyma,  usually  are  incidental  findings.  The 
prevalence  of  palpable  thyroid  nodules  in  the  general 
population  is  4%  in  autopsy  series  50%  is  reached.  95%  of 
thyroid  nodules  are  benign.Williams  Textbook  of 
endocrinology.  Chapter  14.  Nontoxic  Diffuse  and  Nodular 
Goiter and Thyroid Neoplasia.  
 
 
 

Pregunta 205 de 253 
Most common malignant tumor of the thyroid: 

 papillary cancer (10 Puntos) 
 anaplastic cancer 
 Metastasis. 
 follicular cancer 
 medullary cancer 
Correct answer: papillary cancer. Papillary carcinoma is the 
most common (60‐70%) thyroid malignant neoplasm, it has a 
mortality  rate  3‐7%.  Papillary  cancer  is  histologically 
characterized by the presence of pesudopapilas, Psammoma 
bodies,  and  nuclear  grooves  pseudo  inclusions.  Williams 
Textbook  of  endocrinology.  Chapter  14.  Nontoxic  Diffuse 
and Nodular Goiter and Thyroid Neoplasia.  
 
 
 

Pregunta 206 de 253 
The mutation associated with medullary thyroid carcinoma 
is 

 RET (10 Puntos) 
 RAS 
 menina 
 wolframina 
 p53 

Correct answer: RET. Medullary thyroid carcinoma accounts 
for fewer than 10% of thyroid malignancies. It arises from the 
parafollicular or C cells of the thyroid gland, and the tumor 
cells  typically  produce  an  early  biochemical  signal 
(hypersecretion  of  calcitonin).  The  tumor  occurs  in  both 
sporadic  and  hereditary  forms,  the  latter  making  up  about 
20% of the total. The hereditary variety can be transmitted as 
a  single  entity  (familial  MTC),  or  it  can  arise  as  part  of 
MEN2A or MEN2B síndrome associated with the RET proto‐
oncogene. Williams Textbook of endocrinology. Chapter 14. 
Nontoxic  Diffuse  and  Nodular  Goiter  and  Thyroid 
Neoplasia.  
 
 
 

Pregunta 207 de 253 
It is a finding in thyroid ultrasound suggesting malignancy: 

 absence of lymphadenopathy 
 decreased central vascularity 
 eggshell calcifications 
 hyperechogenicity 
 microcalcifications (10 Puntos) 

Correct  answer:  microcalcifications.  Attempts  have  been 


made  to  develop  criteria  for  distinguishing  benign  and 
malignant  nodules.  Echo‐free  (cystic),  spongiform,  and 
homogeneously  hyperechoic  lesions  are  reputed  to  carry  a 
low  risk  of  malignancy.  Positive  predictive  criteria  of 
malignancy  include  predominantly  solid  nodules  and 
absence of cystic elements, hypoechoic nodules, presence of 
microcalcifications,  irregular  margins  and  absence  of  halo, 
and  a  taller  rather  than  a  wide  shape  measured  in  the 
transverse  dimension.  The  color  Doppler  finding  of 
predominantly  internal  or  central  blood  flow  appears  to 
increase  the  risk  that  a  nodule  is  malignant.Williams 
Textbook  of  endocrinology.  Chapter  14.  Nontoxic  Diffuse 
and Nodular Goiter and Thyroid Neoplasia. 
 
 
 
Pregunta 208 de 253 
It is the most aggressive thyroid cancer: 

 follicular 
 lymphoma 
 medullary thyroid carcnioma 
 papillary 
 anaplastic (10 Puntos) 

 
Correct answer: anaplastic. Anaplastic carcinoma constitutes 
about 1% to 2% of all thyroid carcinomas, usually occurs after 
the age of 60 years, and is slightly more common in women. 
This carcinoma is highly malignant, is nonencapsulated, and 
extends widely. Evidence of invasion of adjacent structures, 
such  as  skin,  muscles,  nerves,  blood  vessels,  larynx,  and 
esophagus, is common. Distant metastases occur early in the 
course  of  the  disease  in  lungs,  liver,  bones,  and  brain.  This 
carcinoma  has  the  worst  prognosis  of  thyroid  cancers, 
treatment  should  be  initiated  rapidly  to  avoid  death  from 
locally  infiltrative  disease  and  possible  suffocation. 
Treatment  consists  of  surgical  resection  of  the  tumor  tissue 
present in the neck, if feasible, followed by a combination of 
external irradiation and chemotherapy. Williams Textbook of 
endocrinology.  Chapter  14.  Nontoxic  Diffuse  and  Nodular 
Goiter and Thyroid Neoplasia.  
 
 
 

EHH 
Pregunta 209 de 253 
A 85‐year‐old man, resident of a nursing home is brought to 
the emergency room by decreased alertness, unresponsive to 
stimuli.  He  has  diagnosis  of  diabetes  since  46  years  old, 
treated  with  glyburide  and  metformin.  
Physical  examination:  unconscious,  dehydrated.  BP  90/50, 
CR  105c’,  RR  24x’,  T:  49.7ºC,  bibasilar  pulmonar  crackles. 
Labs: 143 Na, 2.5 K, Gluc 675, pH 7.37. EGO: ketones +, Hb 19, 
Leu  18,500.  
 
 
¿What is the most likely diagnosis?: 

 hyperosmolar hyperglycemic state (10 Puntos) 
 myxedema coma 
 diabetic ketoacidosis 
 cerebral vascular event 
 severe hypoglycemia 

Correct  answer:  hyperosmolar  hyperglycemic  state.HHS  is 


one  of  the  acute  complications  of  diabetes  mellitus.  HHS 
occurs predominantly in type 2 diabetes but can occur in type 
1 most often affects the elderly, nursing home residents and 
people who have free access to fluids. The pathophysiology 
is complex, it is because there is sufficient circulating insulin 
to inhibit lipolysis but is insufficient to induce the peripheral 
utilization  of  glucose. 
characterized  by  severe  hyperglycemia,  hyperosmolality, 
and  dehydration  in  the  absence  of  significant  ketoacidosis. 
Manual de terapéutica médica y procedimientos de urgencias 
INCMSNZ.  Capitulo  35.  Estado  hiperosmolar 
hiperglucémico. 
 
 
 

Pregunta 210 de 253 
The first step in the treatment of this patient is: 

 sodium bicarbonate 
 insulin 
 flumazenil 
 antibiotics 
 hydration (10 Puntos) 

Correct answer: hydration. The mainstay of treatment is fluid 
administration:  SS  0.9  or  0.45%:  2‐3  L  in  the  first  2  h  and 
subsequently SS0.9% 1L every 2 h until blood glucose 250 mg 
/  dL.  Manual  de  terapéutica  médica  y  procedimientos  de 
urgencias  INCMSNZ.  Capitulo  35.  Estado  hiperosmolar 
hiperglucémico. 
 
 
 

Pregunta 211 de 253 
A risk factor for developing this condition is: 

 renal failure 
 glibenclamide using 
 to be resident of a nursing home (10 Puntos) 
 hypokalemia 
 metformin 
Correct  answer:  to  be  resident  of  a  nursing  home. 
It occurs more frequently in elderly, nursing home residents 
or  people  who  are  prone  to  dehydration.  There  are  many 
predisposing  factors,  similar  to  CAD.  To  EHH  the  most 
important  is  dehydration.  Manual  de  terapéutica  médica  y 
procedimientos de urgencias INCMSNZ. Capitulo 35. Estado 
hiperosmolar hiperglucémico. 
 
 
 

Pregunta 212 de 253 
What is the most likely underlying cause of this complication 
in this patient? 

 lack of adherence to treatment 
 hyperglycemia 
 unknown 
 alcoholism 
 infection (10 Puntos) 

Correct answer: infection. The patient is an elderly resident 
of a nursing home, which confers risk of HHS. In addition, 
physical  examination  reveals  the  presence  of  fever  and 
pulmonary rales, suggesting the presence of pneumonia as a 
trigger for HHS. 
 
 
 

Pregunta 213 de 253 
The diagnosis of this complication require the presence of: 
 fever 
 urinary osmolarity> 500 mOsm/l 
 plasma osmolarity >320mOsm/l (10 Puntos) 
 pH <7.30 
 urine ketones +++ 

Correct  answer:  plasma  osmolarity  >320mOsm/l.  The 


diagnostic  criteria  for  HHS  are:  glucose>  600,  pH>  7.30, 
bicarbonate> 18meq / L and plasma osmolality> 320mOsm / 
kg.  Manual  de  terapéutica  médica  y  procedimientos  de 
urgencias  INCMSNZ.  Capitulo  35.  Estado  hiperosmolar 
hiperglucémico. 
 
 
 

Pregunta 214 de 253 
Hombre de 55 años, previamente sano. Antecedentes familiares de 
IAM en su padre a los 45 años y en dos hermanos a los 40 y 50 años.  
Hace 3 meses presentó un episodio de ataque isquémico transitorio, 
se  indicó  tratamiento  con  aspirina  100mg/día.  
EF: TA 130/80, FC 84, FR 19, IMC 24.3. Presencia de arco corneal y 
xantelasma,  cuello  sin  acantosis  nigricans,  ruidos  cardíacos  sin 
alteraciones, xantomas tendinosos. Pulsos de adecuada intensidad. 
Se  realizaron  estudios  de  laboratorio:  glucosa  80,  Cr  0.7, 
triglicéridos150,  colesterol  320,  HDL  45,  LDL  225,  noHDL  275.  
 
 
¿Cuál es el diagnóstico del paciente? 

 Hipertrigliceridemia familiar 
 Hipercolesterolemia familiar (10 Puntos) 
 Hiperlipidemia familiar combinada 
 Dislipidemia secundaria 
 Disbetalipoproteinemia 

 
 
 

Pregunta 215 de 253 
El dato patognomónico de esta enfermedad es: 

 Arco corneal 
 Xantomas tuberosos 
 Xantoma estriata palmaris 
 Xantelasma 
 Xantomas tendinosos (10 Puntos) 

 
 
 

Pregunta 216 de 253 
Además de los cambios en el estilo de vida, usted indica:  

 Atorvastatina (10 Puntos) 
 Niacina  
 Ezetimibe 
 Fenofibrato 
 Insulina 

 
 
 

Pregunta 217 de 253 
¿Cuál es la dispidemia primaria más frecuente en el mundo? 

 Dislipidemia secundaria 
 Disbetalipoproteinemia 
 Hipertrigliceridemia familiar 
 Hipercolesterolemia familiar  
 Hiperlipidemia familiar combinada (10 Puntos) 

 
 
 

Pregunta 218 de 253 
El  dato  carácterístico  en  el  perfil  de  lípidos  que  identifica  a  la 
hipertrigliceridemia familiar es: 

 Relación triglicéridos:colesterol 5:1 (10 Puntos) 
 Relación triglicéridos:colesterol 1:1 
 Relación triglicéridos:colesterol 2:1 
 VLDL elevadas 
 No hay datos característicos 

 
 
 

Pregunta 219 de 253 
Hombre  de  65  años,  de  escasos  recursos  económicos,  alcohólico, 
padece diabetes desde los 46 años, recibe glibenclamida 5mg 2‐1‐1 
y metformina 500mg cada 24hrs, última Hba1c 10%, desde hace 2 
días su familia lo nota soporoso, con lengua seca, fiebre y deterioro 
progresivo,  por  lo  que  decidieron  llevarlo  a  urgencias.  
EF: consciente, reactivo, resequedad de mucosas. TA 90/50 FC 105 
FR 24 T° 39. A la auscultación estertores bibasales, frémitos. Labs: 
Na  143,  K  2.5,  Gluc  675,  pH  7.3.  EGO:  cetonas  ++  
Manejo inicial de este paciente: 

 Todas las anteriores 
 Hidratación (10 Puntos) 
 Insulina 
 Antibióticos 

 
 
 

Pregunta 220 de 253 
Causa desencadenante en este paciente 

 Hiperglicemia 
 Abandono del tratamiento 
 Infección (10 Puntos) 
 Alcoholismo 
 Causa desconocida 

 
 
 
Pregunta 221 de 253 
Complicación  más  frecuente  por  disminuir  la  glucosa 
>100mg/dL/hora  

 Trombosis 
 Infarto 
 Edema cerebral (10 Puntos) 
 Acidosis  
 Hipokalemia  

 
 
 

Pregunta 222 de 253 
Además  de  la  Diabetes,  un  factor  de  riesgo  para  desarrollar  esta 
condición es:  

 Uso de glibenclamida 
 Falla renal 
 Hipokalemia  
 Uso de metformina 
 Alcoholismo (10 Puntos) 

 
 
 

Pregunta 223 de 253 
El requisito de laboratorio para el diagnóstico de esta complicación 
es:  
 Osmolaridad urinaria >500 
 Fiebre 
 pH <7.30 
 Osmolaridad plasmática >320 (10 Puntos) 
 Cetonas +++ en orina 

 
 
 

Pregunta 224 de 253 
Mujer de 22 años, previamente sana, hace 24hrs notó disuria y olor 
fétido  al  orinar,  inició  hace  6  hrs  con  dolor  abdominal,  nausea, 
vómito, cefalea y sensación de falta de aire. A la EF: FC106 FR 32, 
TA  100/60,  palidez  de  tegumentos,  olor  a  cetonas. 
Labs:  Na  132,  K  3,  Mg  1.4  glucosa  400  EGO  pH  6,  leucos  +++, 
bacterias+++, nitritos++, cetonas+++. Gasometria: pH 7.23 HCO3 15 
,  CO2  24,  SatO2  94  
 
Sospecha diagnóstica 

 Descontrol glucemico  
 Estado hiperosmolar  
 Síndrome urémico  
 Cetoacidosis (10 Puntos) 
 Estado hiperosmolar  

 
 
 

Pregunta 225 de 253 
Que laboratorios se requieren 

 Gasometría, glucosa, EGO (10 Puntos) 
 EGO, glucosa, cultivos 
 Gasometría, radiografía de torax, EKG 
 Gasometría, EGO, cultivos 

 
 
 

Pregunta 226 de 253 
Trastorno ácido‐ base presente en esta entidad 

 Alcalosis metabólica  
 Acidosis respiratorio 
 Alcalosis respiratoria  
 Acidosis láctica  
 Acidosis metabólica (10 Puntos) 

 
 
 

Pregunta 227 de 253 
Criterios de resolución en este caso 

 pH 7.3, Gluc < 150, EGO sin cetonas 
 HCO3 >25, CO2 <40, EGO normal. 
 pH 7.4, EKG normal, Na normal 
 pH >7.3, HCO3 >18, Gluc < 200 (10 Puntos) 
 
 
 

Pregunta 228 de 253 
Causa desencadentante de la complicación aguda en este caso 

 Infección (10 Puntos) 
 Edad 
 Diabetes tipo 1 
 Estrés 
 No usar insulina 

 
 
 

Pregunta 229 de 253 
Varón  de  24  años,  que  es  traído  a  urgencias  inconsciente.  Su 
acompañante  refiere  que  hace  3  meses  presentó  un  episodio  de 
síncope,  fue  atendido  en  urgencias  donde  se  administraron 
soluciones  IV  con  mejoría.  Desde  entonces  ha  tenido  múltiples 
episodios de mareo acompañado de sudoración profusa, temblor y 
ansiedad,  que  ceden  al  consumo  de  alimentos  en  general.  Ha 
subido 3 kg debido a que requiere hacer múltiples comidas al día. 
A  la  EF:  paciente  inconsciente,  con  FC  110,  FR  29,  TA  100/60, 
glucemia  capilar  de  30mg/dL. 
 
 
La definición de hipoglucemia es: 

 Glucosa plasmática <100mg/dL 
 Glucosa plasmática <70mg/dL 
 Glucosa plasmática <90mg/dL 
 Glucosa plasmática <85mg/dL 
 Glucosa plasmática <55mg/dL (10 Puntos) 

 
 
 

Pregunta 230 de 253 
La primera medida a realizar en este caso es: 

 Intubación endotraqueal 
 Colocar  sonda  nasogástrica  y  administrar  solución  glucosada 
50%. 
 Hidratación con SS0.9% 
 Administrar 1 mg de glucagón (10 Puntos) 
 Administrar 15g de glucosa VO (jugo) 

 
 
 

Pregunta 231 de 253 
Son  síntomas  adrenérgicos  de  hipoglucemia  todos  los  siguientes, 
excepto:  

 Sudoración 
 Temblor 
 Convulsiones (10 Puntos) 
 Ansiedad 
 Palidez  

 
 
 

Pregunta 232 de 253 
De los siguientes fármacos cual NO se asocia a hipoglucemia  

 Insulina glargina 
 Metformina (10 Puntos) 
 Insulina rápida 
 Sulfonilureas 
 Sitagliptina  

 
 
 

Pregunta 233 de 253 
Es una causa de hiperinsulinismo endógeno: 

 Insulinoma (10 Puntos) 
 Anticuerpos anti‐GAD 
 Enfermedad de Cushing 
 Feocromocitoma 
 Insuficiencia hepática 

 
 
 
Pregunta 234 de 253 
Hombre de 50 años, con obesidad, hipertensión arterial sistémica, 
y  diabetes  tipo  2  desde  hace  15  años.  En  tratamiento  con 
metformina  500mg  cada  12h  y  glibenclamida  5mg  cada  12h, 
enalapril 10mg cada 24h. Refiere edema de miembros inferiores de 
predominio  vespertino.  Última  HbA1c  de  9%.  
Acude por presentar confusión y dificultad para hablar de 15 min 
de  evolución.  
EF:  FC  100lpm,  FR  22rpm,  TA  140/90,  pálido,  diaforético,  con 
edema  hasta  tercio  inferior  de  ambas  piernas,  con  godete. 
Laboratorios:  Glucosa  44mg/dL,  Creatinina  0.9  mg/dL.  
 
¿Cuál es la causa de hipoglucemia en este caso? 

 Mal apego al tratamiento 
 Glibenclamida (10 Puntos) 
 Metformina  
 Falla renal 
 Enalapril. 

 
 
 

Pregunta 235 de 253 
El mecanismo de acción de la glibenclamida es: 

 Cierra  los  canales  de  K  sensibles  a  ATP  en  la  célula  beta  (10 
Puntos) 
 Inhibe la DPP4 
 Suprime la gluconeogénesis hepática 
 Inhibe a la alfa‐glucosidasa 
 Inhibe la despolarización de la célula beta 

 
 
 

Pregunta 236 de 253 
El tratamiento indicado para la hipoglucemia en este caso es: 

 Administrar 15g de glucosa VO (jugo) (10 Puntos) 
 Solución glucosada al 5% 
 Administrar 1 mg de glucagón  
 Hidrocortisona 50mg IV 
 Solución glucosada al 50% 

 
 
 

Pregunta 237 de 253 
Son síntomas neuroglucopénicos todos los siguientes, excepto:  

 Mareos 
 Convulsiones 
 Ansiedad (10 Puntos) 
 Confusión 
 Parestesias 

 
 
 
Pregunta 238 de 253 
La ausencia de síntomas adrenérgicos en este paciente es debida a:  

 Los  síntomas  adrenérgicos  solamente  se  dan  con  la 


administración exógena de insulina 
 El nivel de glucosa no era <40 
 Ausencia  de  respuesta  de  glucagón  por  diabetes  de  larga 
evolución 
 Reducción de la respuesta de epinefrina por diabetes de larga 
evolución. (10 Puntos) 

 
 
 

Pregunta 239 de 253 
Hombre  de  26  años,  cuenta  con  el  diagnóstico  de  diabetes  tipo  1 
desde hace 7 años, fumador. Ha estado hospitalizado dos veces por 
crisis  hiperglucémicas.  No  se  han  estudiado  complicaciones 
crónicas.  Refiere  desde  hace  algunos  meses  parestesias  en  las 
plantas de los pies y dolor ardoroso. Utiliza insulina NPH 20UI en 
la  mañana  y  10  UI  en  la  noche.  
Laboratorios  recientes:  Hba1c  de  9.8%,  glucosa  270mg/dl, 
creatinina  0.88  mg/dl,  colesterol  215  mg/dL,  HDL  38,  LDL  121 
mg/dL,  triglicéridos  190  mg/dL.  Albumina  en  orina  de  24  horas: 
10mg/día. 
 
¿Cuál es la complicación aguda más frecuente en esta enfermedad? 

 Neumonía 
 Cetoacidosis diabética. (10 Puntos) 
 Infección urinaria 
 Pancreatitis 
 Estado hiperosmolar hiperglucémico 

 
 
 

Pregunta 240 de 253 
¿Cuándo  se  recomienda  la  primera  revisión  oftalmológica 
endiabetes tipo 1?  

 A los 3 años del diagnóstico 
 No se requiere revisión oftalmológica 
 Al momento del diagnóstico 
 A los 5 años del diagnóstico (10 Puntos) 
 A los 10 años del diagnóstico  

 
 
 

Pregunta 241 de 253 
La fisiopatogenia de esta enfermedad es:  

 Secundaria a isquemia 
 Secundaria a destrucción autoinmune del páncreas (10 Puntos) 
 Secundaria a una mutación inactivante del receptor de insulina.  
 Secundaria a mala alimentación 
 Secundario a una infección  

 
 
 

Pregunta 242 de 253 
Como meta de prevención primaria, este paciente requiere:  

 Insulina ultralenta 
 Marcapasos  
 Fibrato  
 Estatina (10 Puntos) 
 IECA o ARA  

 
 
 

Pregunta 243 de 253 
Diabetes  insulinodependiente  que  por  lo  general  se  presenta 
después de los 25 años y tiene etiología autoinmune:  

 Diabetes tipo 2 
 Diabetes por daño pancreático 
 Diabetes tipo LADA (10 Puntos) 
 Diabetes tipo MODY 
 Diabetes insípida.  

 
 
 

Pregunta 244 de 253 
Hombre de 62 años con una diabetes mellitus tipo 2 de 10 años de 
evolución,  en  tratamiento  con  metformina  y  sitagliptina.  Hace 
ejercicio físico escaso y realiza una dieta adecuada. En los últimos 
6 meses ha perdido peso y tiene astenia. Sus controles glucémicos 
se han deteriorado pasando de glucemias basales de 110‐140 mg/dl 
a  glucemias  de  170‐200  mg/dl,  así  como  su  hemoglobina 
glucosilada que ha pasado de 7,1 a 8,5%. Pesa 60kg, mide 1.67. TA 
120/70. 
 
La medida terapéutica más adecuada a realizar es: 

 Aumentar  la  ingesta  de  proteínas  e  hidratos  de  carbono  de 


cadena larga en la dieta para mejorar la astenia y la pérdida de peso 
 Sustituir sitagliptina por pioglitazona 
 Asociar al tratamiento ascarbosa. 
 Asociar al tratamiento una dosis de insulina basal. (10 Puntos) 
 Sustituir metformina por glimepirida 

 
 
 

Pregunta 245 de 253 
La causa de fracaso al tratamiento es: 

 Mal apego al tratamiento farmacológico 
 Mal apego a dieta 
 Pérdida de receptores para sitagliptina 
 Pérdida de la función de las células beta (10 Puntos) 
 Deterioro renal 

 
 
 

Pregunta 246 de 253 
El esquema de insulina que elegiría para este paciente sería:  

 Bolos preprandiales de insulina aspart 
 Basal – bolos con insulina NPH y rápida 
 No indicaría insulina 
 Dosis única de NPH o glargina. (10 Puntos) 
 Bolos preprandiales de insulina regular 

 
 
 

Pregunta 247 de 253 
De  las  siguientes  insulinas,  elija  la  que  posee  el  efecto  más 
prolongado: 

 NPH 
 Lispro  
 Regular 
 Glargina (10 Puntos) 
 Glulisina  

 
 
 

Pregunta 248 de 253 
De las siguientes insulinas, elija la que posee el efecto más corto:  

 Glargina 
 Lispro (10 Puntos) 
 NPH 
 Detemir 
 Regular 

 
 
 

Pregunta 249 de 253 
Hombre de 14 años, sin antecedentes familiares de diabetes u otras 
enfermedades  de  importancia.  
Se estableció el diagnóstico de Diabetes mellitus tipo 1 a los 13 años 
de  edad,  con  glucemia  al  azar  >600  con  signos  y  síntomas 
compatibles.  Tiene  positividad  para  Ac  anti  GAD  en  varias 
mediciones,  asimismo  determinación  de  péptido  C  bajo.  
Inicialmente fue tratado con insulina NPH 10 UI en la mañana y 5 
UI  en  la  noche,  así  como  insulina  regular  3  UI  preprandiales. 
Realiza 1 hora de ejercicio diariamente y sigue un plan de 1700 Kcal, 
con  40%  de  carbohidratos.  
Un  año  después  del  diagnóstico  presentó  hipoglucemias 
frecuentes,  por  lo  que  se  redujo  progresivamente  la  dosis  de 
insulina  hasta  suspenderla,  sin  tratamiento  desde  hace  3  meses. 
Glucemias preprandiales 100‐110 y posprandiales 120‐140, HbA1c 
6%.  
El cuadro del paciente se explica por lo siguiente  

 El diagnóstico inicial de DM1 fue incorrecto 
 El diagnóstico del paciente es DM2 y mejoró con dieta y ejercicio  
 Se trata de una diabetes tipo LADA 
 Presentó DM1 transitoria, que se ha resuelto 
 Se encuentra en fase de luna de miel (10 Puntos) 

 
 
 

Pregunta 250 de 253 
El  porcentaje  de  carbohidratos  que  debe  incluir  la  dieta  de  un 
paciente con diabetes es:  

 30‐40% 
 10‐20% 
 20‐30% 
 40‐50% (10 Puntos) 
 60% 

 
 
 

Pregunta 251 de 253 
Es un factor de riesgo para presentar diabetes tipo 1: 

 Tabaquismo 
 Antecedentes familiares de autoinmunidad (10 Puntos) 
 Obesidad 
 Radiación in utero 
 Consumo de drogas  

 
 
 

Pregunta 252 de 253 
¿Qué porcentaje de casos de diabetes son tipo 1? 

 1% 
 20% 
 50% 
 90% 
 5‐10% (10 Puntos) 

 
 
 

Pregunta 253 de 253 
¿Cuáles  son  los  anticuerpos  más  útiles  para  el  diagnóstico  de 
diabetes tipo 1? 

 Anti – IA2 
 Anti‐ nucleares 
 Anti‐ descarboxilasa del ácido glutámico (10 Puntos) 
 Anti ‐ proteosoma 
 Anti‐ insulina 

También podría gustarte